HESI Exit Exam Questions

Study with our free HESI Exit Exam Questions, Qbank Test 1. The questions include answers and detailed explanations. HESI Exit nursing topics focus on critical thinking, priorities of care, health promotion and maintenance, safe and effective care, and basic care and comfort, treatments and nursing management. Achieve your best grade on the HESI Exit Exam!
HESI Exit Exam 2
HESI Exit Exam 3
HESI Exit Exam 4
HESI Exit Exam 5
HESI Exit Exam 6
HESI Exit Exam 7
HESI Exit Exam 8
HESI Exit Exam 9
HESI Exit Exam 10

HESI Exit Question 1: Which of the following cardiac disorders is characterized by the accumulation of fluid in the pericardial sac, leading to impaired cardiac function?
A) Atrial fibrillation
B) Myocardial infarction
C) Pericardial effusion
D) Ventricular tachycardia

Explanation: Pericardial effusion is the accumulation of fluid in the pericardial sac, which surrounds the heart. This excess fluid can compress the heart, impairing its ability to fill and pump blood effectively. Atrial fibrillation, myocardial infarction, and ventricular tachycardia are cardiac disorders, but they do not involve fluid accumulation around the heart.

POP QUIZ Question

@qbankproacademy

4000+ Free NCLEX QUESTIONS. Go to QBankProAcademy.com FREE Qbank questions for NCLEX RN, PN, HESI Exit, Med Surg, AANP, ANCC, and HesiA2. Practice questions, quizzes, and listen to the Free Podcast. In this video, we review a question on MEDICATION ADMINISTRATION in Nursing Care. Free nursing NCLEX 60-DAY CHALLENGE https://qbankpro.thinkific.com/courses/qbankpro-academy At QbankproAcademy.com our mission is to provide free QBanks, videos, and the most up to date test prep information for nurses. If you find our website helpful, please tell other aspiring nurses, nursing students, and professors. Please link to our site from your blogs, videos, and college websites, or share us on your favorite social media sites. Thank you for your support! nclex Nurses Nursing aanp qbank ancc hesi medsurg qbank

♬ original sound – QBank Pro Academy – QBank Pro Academy

HESI Exit Question 2: A 57-year-old female is admitted to the Emergency Room with severe chest pain (9/10) radiating down the left arm. The pain began 1.5 hours ago. She reports feeling nauseated and shortness of breath. Her vital signs: T- 98.7 F, heart rate- 99; RR- 20 bpm; B/P- 140/95; oximeter- 92% on room air. Labs and ECG are:

Cholesterol: 250 mg/dl
WBC: 8 x 10^3 cells/mm3
Platelets: 430,000/ mm3
Potassium: 3.6 mEq/L
Sodium: 140 mEq/L
Magnesium: 1.4 mEq/L
Troponin: 0.5 ng/ml
Creatine Kinase MB: 150 IU/L
ECG: ST segment elevation in the anterior leads, sinus rhythm, 99 bpm

What are the two most significant findings?
Cholesterol
ECG
WBC
Troponin
Potassium
Calcium

Explanation: The presentation is typical for myocardial infarction (MI). The ECG with ST segment elevation indicates there is injury to the heart muscle and must be addressed immediately. Elevated troponin also indicates myocardial injury has occurred recently. The other abnormal lab findings are notable and will be addressed but they are not as significant or diagnostic for MI.

NCLEX Practice: 200+ Quizzes

HESI Exit Question 3: Which cardiac disorder is characterized by a chaotic, irregular heartbeat with no discernible P waves on an ECG, often described as “quivering” of the atria?
A) Atrial fibrillation
B) Ventricular fibrillation
C) Supraventricular tachycardia
D) Bradycardia

Explanation: Atrial fibrillation (AFib) is characterized by an irregular, rapid heartbeat with no discernible P waves on an ECG. It can lead to an increased risk of stroke and other complications. Ventricular fibrillation, supraventricular tachycardia, and bradycardia are different cardiac rhythm disorders.

atrial fibrillation, fnp, agnp, exam question, ancc,aanp, nclex, hesi
Atrial fibrillation showing an irregular rhythm

HESI Exit Question 4: In which cardiac disorder does the heart muscle become abnormally thick, making it harder for the heart to pump blood effectively?
A) Pericarditis
B) Cardiomyopathy
C) Aortic stenosis
D) Arrhythmia

Explanation: Cardiomyopathy is a condition where the heart muscle becomes abnormally thick, stiff, or weak, making it difficult for the heart to pump blood effectively. The other options are cardiac disorders, but they do not specifically involve thickening of the heart muscle.

HESI Exit Question 5: Which cardiac disorder involves the narrowing of the aortic valve, restricting the flow of blood from the left ventricle to the aorta?
A) Heart failure
B) Aortic insufficiency
C) Aortic stenosis
D) Ventricular septal defect

Explanation: Aortic stenosis is a condition where the aortic valve narrows, reducing the flow of blood from the left ventricle to the aorta. This can lead to various symptoms, including chest pain and shortness of breath. The other options are cardiac disorders, but they do not specifically involve aortic valve narrowing.

 
FREE nclex practice questions
Click “Join Us”: Become a Member – QBankProAcademy.com

HESI Exit Question 6: Which cardiac disorder is characterized by the inability of the heart to pump blood effectively, leading to fluid retention, fatigue, and shortness of breath?
A) Atrial fibrillation
B) Myocardial infarction
C) Congestive heart failure
D) Ventricular fibrillation

Explanation: Congestive heart failure (CHF) occurs when the heart is unable to pump blood effectively, leading to a buildup of fluid in the lungs and other parts of the body. This results in symptoms like fatigue, shortness of breath, and fluid retention. The other options are cardiac disorders, but they do not specifically involve heart failure.

HESI Exit Question 7: Which cardiac disorder is characterized by a sudden, rapid heartbeat that originates above the ventricles and may cause palpitations, dizziness, or fainting?
A) Atrial fibrillation
B) Ventricular fibrillation
C) Supraventricular tachycardia
D) Bradycardia

Explanation: Supraventricular tachycardia (SVT) is a rapid heartbeat that originates above the ventricles. It can lead to palpitations, dizziness, and fainting. The other options are cardiac rhythm disorders, but they have different characteristics.

HESI Exit Question 8: Which cardiac disorder is characterized by the formation of fatty deposits or plaques in the coronary arteries, leading to reduced blood flow to the heart muscle?
A) Angina pectoris
B) Atrial fibrillation
C) Pericardial effusion
D) Cardiomyopathy

Explanation: Angina pectoris is characterized by chest pain or discomfort due to reduced blood flow to the heart muscle caused by the buildup of fatty deposits in the coronary arteries. The other options are cardiac disorders, but they do not specifically involve plaque formation in the arteries.

HESI Exit Question 9: Which cardiac disorder is characterized by a rapid, irregular heartbeat originating in the ventricles, often leading to loss of consciousness and requiring immediate defibrillation?
A) Atrial fibrillation
B) Ventricular fibrillation
C) Supraventricular tachycardia
D) Bradycardia

Explanation: Ventricular fibrillation (VFib) is a life-threatening arrhythmia characterized by rapid, irregular ventricular contractions. It often leads to loss of consciousness and requires immediate defibrillation to restore a normal heart rhythm. The other options are different types of cardiac rhythm disorders.

Ventricular Fibrillation, nclex, AANP, ANCC, Questions
“VFib”

HESI Exit Question 10: Which cardiac disorder is characterized by the inflammation of the inner lining of the heart chambers and heart valves, often associated with infections?
A) Endocarditis
B) Myocarditis
C) Atrial fibrillation
D) Aortic stenosis

Explanation: Endocarditis is characterized by the inflammation of the inner lining of the heart chambers and valves, typically associated with bacterial or fungal infections. The other options are cardiac disorders, but they do not specifically involve inflammation of the endocardium.

Do more cardiovascular questions

HESI Exit Question 11: A 55-year-old male presents to the emergency department with sudden-onset leg pain, pallor, and coolness. He has a history of smoking and hypertension. On examination, his left leg appears pale, and there is a complete absence of pulses in the left femoral and popliteal arteries. What is the most likely diagnosis?
A) Deep vein thrombosis (DVT)
B) Arterial embolism
C) Peripheral artery disease (PAD)
D) Compartment syndrome

Rationale: The sudden onset of leg pain, pallor, coolness, and absence of pulses in the presence of risk factors such as smoking and hypertension is highly suggestive of an arterial embolism. Arterial embolism is the occlusion of an artery by an embolus, often originating from the heart or a large artery. It can lead to acute limb ischemia.

HESI Exit Question 12: A 65-year-old female presents with intermittent claudication, characterized by leg pain during walking that is relieved with rest. She has a history of diabetes mellitus and hyperlipidemia. Which condition is most likely causing her symptoms?
A) Deep vein thrombosis (DVT)
B) Arterial embolism
C) Peripheral artery disease (PAD)
D) Venous insufficiency

Rationale: Intermittent claudication, which is leg pain during walking relieved by rest, is a classic symptom of peripheral artery disease (PAD). PAD is commonly seen in patients with risk factors such as diabetes mellitus and hyperlipidemia.

HESI Exit Question 13: A 60-year-old male with a history of atrial fibrillation is prescribed warfarin (Coumadin) to prevent clot formation. Which laboratory test should be monitored regularly to assess the effectiveness of warfarin therapy?
A) Complete blood count (CBC)
B) International normalized ratio (INR)
C) Prothrombin time (PT)
D) Activated partial thromboplastin time (aPTT)

Rationale: Warfarin (Coumadin) therapy is monitored by assessing the International Normalized Ratio (INR). INR measures the effectiveness of warfarin in preventing excessive blood clotting and helps ensure that the patient is within the therapeutic range.

HESI Exit Question 14: A 45-year-old female presents with calf pain, swelling, and redness in her left leg. She has a family history of deep vein thrombosis (DVT). Which diagnostic test is commonly used to confirm the presence of DVT?
A) Magnetic resonance imaging (MRI)
B) Computed tomography (CT) scan
C) Venous Doppler ultrasound
D) Arteriography

Rationale: Venous Doppler ultrasound is a common diagnostic test for confirming the presence of deep vein thrombosis (DVT). It is non-invasive and can visualize blood flow in the deep veins, helping to identify clots.

HESI Exit Question 15: A 70-year-old male with a history of hypertension and atrial fibrillation is prescribed a direct oral anticoagulant (DOAC) for stroke prevention. Which medication is likely prescribed to this patient?
A) Clopidogrel (Plavix)
B) Warfarin (Coumadin)
C) Dabigatran (Pradaxa)
D) Aspirin

Rationale: Direct oral anticoagulants (DOACs) are commonly prescribed for patients with atrial fibrillation for stroke prevention. Dabigatran (Pradaxa) is one of the DOACs used for this purpose.

HESI Exit Question 16: A 62-year-old male presents with sudden-onset chest pain and shortness of breath. He has a history of smoking and hypertension. An ECG shows ST-segment elevation in leads II, III, and aVF. What is the most likely diagnosis?
A) Aortic dissection
B) Myocardial infarction
C) Pulmonary embolism
D) Pericarditis

Rationale: ST-segment elevation in an ECG, along with symptoms of chest pain and shortness of breath, is highly indicative of a myocardial infarction (heart attack). This patient is experiencing an ST-segment elevation myocardial infarction (STEMI).

HESI Exit Question 17: A 58-year-old female presents with swollen, painful legs, and skin changes in the lower extremities. She reports a long history of standing for extended periods at her job. What is the most likely diagnosis?
A) Deep vein thrombosis (DVT)
B) Arterial embolism
C) Peripheral artery disease (PAD)
D) Chronic venous insufficiency

Rationale: The patient’s history of prolonged standing, along with swollen, painful legs and skin changes, suggests chronic venous insufficiency. This condition results from damaged valves in the veins, leading to poor venous return.

HESI Exit Question 18: A 70-year-old male with a history of aortic aneurysm is prescribed a medication that reduces blood pressure by blocking the effects of angiotensin II. Which medication is he most likely taking?
A) Amlodipine (Norvasc)
B) Metoprolol (Lopressor)
C) Lisinopril (Prinivil)
D) Furosemide (Lasix)

Rationale: Lisinopril is an angiotensin-converting enzyme (ACE) inhibitor commonly prescribed to reduce blood pressure and prevent complications in patients with aortic aneurysms.

Transabdominal ultrasound of abdominal aortic aneurysm (AAA), NCLEX, ANCC, AANP, HESI EXIT, HESI A2
Transabdominal ultrasound of abdominal aortic aneurysm (AAA)

HESI Exit Question 19: A 50-year-old male presents with sudden, severe chest pain and shortness of breath. He is diaphoretic, and his blood pressure is significantly elevated. An ECG shows ST-segment elevation in the anterior leads. What is the most likely diagnosis?
A) Pulmonary embolism
B) Aortic dissection
C) Myocardial infarction
D) Pericarditis

Rationale: The combination of severe chest pain, diaphoresis, elevated blood pressure, and ST-segment elevation on ECG indicates a myocardial infarction. This patient is experiencing a non-ST-segment elevation myocardial infarction (NSTEMI).

HESI Exit Question 20: A 65-year-old female presents with leg pain, swelling, and a history of varicose veins. She complains of pain that worsens with prolonged standing and is relieved when she elevates her legs. What is the most likely diagnosis?
A) Deep vein thrombosis (DVT)
B) Arterial embolism
C) Peripheral artery disease (PAD)
D) Venous insufficiency

Rationale: The patient’s history of varicose veins, leg pain worsened by prolonged standing, and relief with leg elevation are indicative of venous insufficiency. This condition is often associated with damaged venous valves, leading to poor blood return to the heart.

HESI Exit Question 21: A 45-year-old female presents with a chronic cough, dyspnea, and wheezing. She reports a history of smoking and exposure to environmental pollutants. Which respiratory disorder is most likely responsible for her symptoms?
A) Pneumonia
B) Chronic obstructive pulmonary disease (COPD)
C) Pulmonary embolism
D) Tuberculosis

Rationale: The patient’s chronic cough, dyspnea, wheezing, history of smoking, and exposure to environmental pollutants are indicative of COPD, a chronic respiratory disorder commonly caused by smoking and environmental factors.

HESI Exit Question 22: A 60-year-old male presents with sudden-onset pleuritic chest pain and dyspnea. On auscultation, you hear absent breath sounds on the left side and tracheal deviation to the right. What condition is most likely causing these findings?
A) Pneumothorax
B) Pulmonary embolism
C) Pleurisy
D) Asthma

Rationale: The sudden-onset pleuritic chest pain, absent breath sounds, and tracheal deviation to the right are typical findings in a tension pneumothorax, where air accumulates in the pleural space and compresses the lung.

HESI Exit Question 23: A 30-year-old male presents with a productive cough, fever, and chest pain. A chest X-ray reveals a dense consolidation in the right lower lobe. What is the most likely diagnosis?
A) Tuberculosis
B) Pneumonia
C) Bronchitis
D) Lung cancer

Rationale: The patient’s symptoms, along with the chest X-ray findings of dense consolidation in a specific lung lobe, are suggestive of pneumonia, an infection of the lung tissue.

HESI Exit Question 24: A 50-year-old female with a history of asthma presents with acute exacerbation. She is using accessory muscles for breathing and has a prolonged expiratory phase. What medication is indicated as a first-line treatment in this situation?
A) Albuterol (Ventolin)
B) Montelukast (Singulair)
C) Ipratropium (Atrovent)
D) Prednisone

Rationale: In an acute asthma exacerbation, a short-acting beta-agonist like albuterol is indicated as a first-line treatment to relieve bronchoconstriction and improve airflow.

DYSPNEA, inhaler, asthma, NCLEX, ANCC, AANP, HESI EXIT, HESI A2
Use of an inhaler for shortness of breath in an asthmatic.

HESI Exit Question 25: A 55-year-old male presents with hemoptysis (coughing up blood), chest pain, and weight loss. A chest X-ray shows a cavitary lesion in the right upper lobe. What condition is most likely causing these findings?
A) Tuberculosis
B) Pneumonia
C) Bronchitis
D) Lung cancer

Rationale: The combination of hemoptysis, chest pain, weight loss, and a cavitary lesion on chest X-ray is highly suggestive of lung cancer.

HESI Exit Question 26: A 40-year-old female presents with sudden-onset pleuritic chest pain and dyspnea. She recently had surgery and has been immobile. What condition should be ruled out in this patient?
A) Pneumothorax
B) Pulmonary embolism
C) Pleurisy
D) Asthma

Rationale: Sudden-onset pleuritic chest pain and dyspnea in a patient who has recently undergone surgery and been immobile raise concern for a pulmonary embolism, which is a clot in the lung arteries.

HESI Exit Question 27: A 65-year-old male with a history of chronic heart failure presents with dyspnea, orthopnea, and crackles in the lung bases. What respiratory medication is commonly prescribed to manage these symptoms?
A) Albuterol (Ventolin)
B) Montelukast (Singulair)
C) Furosemide (Lasix)
D) Prednisone

Rationale: Furosemide, a loop diuretic, is commonly prescribed to manage fluid overload in patients with chronic heart failure, which can manifest as dyspnea and crackles in the lungs.

HESI Exit Question 28: A 35-year-old male presents with acute, severe dyspnea, tachycardia, and hypoxemia. He reports a recent long-haul flight. What condition should be suspected in this patient?
A) Pneumothorax
B) Pulmonary embolism
C) Pleurisy
D) Asthma

Rationale: Acute dyspnea, tachycardia, and hypoxemia in a patient with a history of recent travel, especially a long-haul flight, are highly suggestive of a pulmonary embolism.

HESI Exit Question 29: A 50-year-old female with a history of chronic obstructive pulmonary disease (COPD) experiences frequent exacerbations. Which respiratory medication class is often used to reduce exacerbations in patients with COPD?
A) Short-acting beta-agonists (SABAs)
B) Anticholinergics
C) Inhaled corticosteroids (ICS)
D) Leukotriene modifiers

Rationale: Anticholinergic medications, such as tiotropium, are often used to reduce exacerbations and improve lung function in patients with COPD.

HESI Exit Question 30: A 45-year-old male presents with a chronic, non-productive cough, clubbing of the fingers, and fine inspiratory crackles on auscultation. What condition is most likely responsible for these findings?
A) Tuberculosis
B) Pneumonia
C) Bronchitis
D) Idiopathic pulmonary fibrosis (IPF)

Rationale: The chronic non-productive cough, clubbing of the fingers, and fine inspiratory crackles are typical findings in idiopathic pulmonary fibrosis (IPF), a progressive interstitial lung disease.

HESI Exit Question 31: A 60-year-old female presents with sudden-onset, severe chest pain, pleuritic in nature, and worsened with deep inspiration. She reports a history of recent surgery and immobility. What condition should be suspected in this patient?
A) Pneumothorax
B) Pulmonary embolism
C) Pleurisy
D) Asthma

Rationale: Sudden-onset pleuritic chest pain worsened with deep inspiration in a patient with a history of recent surgery and immobility suggests pleurisy, inflammation of the pleura.

HESI Exit Question 32: A 50-year-old male with a history of asthma experiences symptoms despite using a short-acting beta-agonist (SABA) as needed. What type of respiratory medication is commonly added for long-term control in this patient?
A) Inhaled corticosteroids (ICS)
B) Long-acting beta-agonists (LABAs)
C) Anticholinergics
D) Leukotriene modifiers

Rationale: Inhaled corticosteroids (ICS) are often added for long-term control in patients with persistent asthma symptoms, even when using SABAs as needed.

HESI Exit Question 33: A 55-year-old male with a history of chronic heart failure presents with nocturnal dyspnea and paroxysmal nocturnal dyspnea (PND). What respiratory medication may be prescribed to relieve these symptoms?
A) Albuterol (Ventolin)
B) Montelukast (Singulair)
C) Furosemide (Lasix)
D) Prednisone

Rationale: Furosemide is a loop diuretic commonly prescribed to manage fluid overload and relieve symptoms like nocturnal dyspnea and PND in patients with heart failure.

HESI Exit Question 34: A 70-year-old female presents with chronic cough, sputum production, and a history of exposure to dust and fumes. What condition is most likely responsible for her symptoms?
A) Tuberculosis
B) Pneumonia
C) Bronchitis
D) Chronic obstructive pulmonary disease (COPD

Rationale: The chronic cough, sputum production, and history of dust and fume exposure are indicative of COPD, a chronic respiratory condition often associated with occupational exposures.

HESI Exit Question 35: A 65-year-old male with a history of hypertension is prescribed a medication that relaxes the smooth muscles of the airways to improve airflow. Which respiratory medication is he most likely taking?
A) Albuterol (Ventolin)
B) Montelukast (Singulair)
C) Ipratropium (Atrovent)
D) Prednisone

Rationale: Albuterol is a short-acting beta-agonist (SABA) that relaxes the smooth muscles of the airways, improving airflow. It is commonly used to treat bronchoconstriction in conditions like asthma and COPD.

HESI Exit Question 36: A 50-year-old female presents with a persistent, dry cough, fatigue, and shortness of breath. A chest X-ray reveals bilateral hilar lymphadenopathy. What condition should be suspected in this patient?
A) Tuberculosis
B) Pneumonia
C) Sarcoidosis
D) Lung cancer

Rationale: The combination of persistent dry cough, fatigue, shortness of breath, and bilateral hilar lymphadenopathy is suggestive of sarcoidosis, a multisystem inflammatory disorder that often affects the lungs.

HESI Exit Question 37: A 40-year-old male with a history of smoking presents with a chronic, productive cough, and dyspnea. On auscultation, you hear coarse crackles in the lung bases. What condition is most likely responsible for these findings?
A) Tuberculosis
B) Pneumonia
C) Bronchitis
D) Chronic bronchitis

Rationale: Chronic bronchitis is characterized by a chronic productive cough, dyspnea, and coarse crackles in the lung bases. It is often seen in patients with a history of smoking.

HESI Exit Question 38: A 55-year-old male with a history of allergic rhinitis and nasal congestion is prescribed a medication that reduces nasal congestion and improves airflow. Which respiratory medication is he most likely taking?
A) Albuterol (Ventolin)
B) Montelukast (Singulair)
C) Ipratropium (Atrovent)
D) Pseudoephedrine (Sudafed)

Rationale: Pseudoephedrine is a nasal decongestant commonly used to reduce nasal congestion and improve airflow in conditions like allergic rhinitis.

HESI Exit Question 39: A 60-year-old male presents with progressive dyspnea, bibasilar crackles, and a non-productive cough. A chest X-ray reveals “honeycombing” in the lung parenchyma. What condition is most likely responsible for these findings?
A) Tuberculosis
B) Pneumonia
C) Bronchitis
D) Idiopathic pulmonary fibrosis (IPF)

Rationale: The combination of progressive dyspnea, bibasilar crackles, non-productive cough, and “honeycombing” on chest X-ray is characteristic of idiopathic pulmonary fibrosis (IPF), a progressive interstitial lung disease.

HESI Exit Question 41: A 65-year-old male presents with severe joint pain and stiffness, which is worse in the morning and lasts for more than 30 minutes. He also reports small, bony growths on his fingers. What condition is most likely causing these symptoms?
A) Osteoarthritis
B) Rheumatoid arthritis
C) Gout
D) Ankylosing spondylitis

Rationale: The patient’s symptoms of morning stiffness lasting over 30 minutes, joint pain, and the presence of bony growths (rheumatoid nodules) on the fingers are indicative of rheumatoid arthritis.

HESI Exit Question 42: A 40-year-old female presents with severe lower back pain and stiffness that improves with movement. She also mentions experiencing morning stiffness. What condition should be suspected in this patient?
A) Osteoarthritis
B) Rheumatoid arthritis
C) Gout
D) Ankylosing spondylitis

Rationale: The patient’s lower back pain, morning stiffness, and improvement of symptoms with movement are characteristic of ankylosing spondylitis, a type of inflammatory arthritis that primarily affects the spine.

HESI Exit Question 43: A 55-year-old male presents with sudden-onset, severe shoulder pain and limited range of motion. He reports a history of diabetes. What condition is most likely causing these symptoms?
A) Rotator cuff tear
B) Frozen shoulder (adhesive capsulitis)
C) Osteoarthritis
D) Gout

Rationale: The patient’s sudden-onset shoulder pain and limited range of motion are indicative of frozen shoulder (adhesive capsulitis), which is more common in individuals with diabetes.

HESI Exit Question 44: A 60-year-old female presents with joint pain, swelling, and redness in her hands, especially in the small joints of her fingers. What condition is most likely causing these findings?
A) Osteoarthritis
B) Rheumatoid arthritis
C) Gout
D) Ankylosing spondylitis

Rationale: The patient’s joint pain, swelling, redness, and involvement of the small joints in the fingers are characteristic of rheumatoid arthritis.

HESI Exit Question 45: A 45-year-old male presents with sudden-onset, severe pain and swelling in his big toe. He reports a history of a high-protein diet and alcohol consumption. What condition is most likely causing these symptoms?
A) Osteoarthritis
B) Rheumatoid arthritis
C) Gout
D) Ankylosing spondylitis

Rationale: The patient’s sudden-onset pain and swelling in the big toe, along with a history of a high-protein diet and alcohol consumption, are suggestive of gout, a type of inflammatory arthritis.

HESI Exit Question 46: A 70-year-old female presents with chronic hip pain, stiffness, and limited range of motion. X-rays reveal joint space narrowing and osteophyte formation. What condition is most likely causing these findings?
A) Osteoarthritis
B) Rheumatoid arthritis
C) Gout
D) Ankylosing spondylitis

Rationale: The patient’s chronic hip pain, joint space narrowing, and osteophyte formation on X-rays are indicative of osteoarthritis.

HESI Exit Question 47: A 55-year-old male presents with pain, swelling, and stiffness in multiple joints, including the knees and wrists. He reports a history of psoriasis. What condition should be suspected in this patient?
A) Osteoarthritis
B) Rheumatoid arthritis
C) Psoriatic arthritis
D) Gout

Rationale: The patient’s joint symptoms, along with a history of psoriasis, are characteristic of psoriatic arthritis, a type of inflammatory arthritis associated with psoriasis.

HESI Exit Question 48: A 60-year-old female presents with chronic, widespread muscle pain and tenderness. She also reports sleep disturbances and fatigue. What condition is most likely causing these symptoms?
A) Osteoarthritis
B) Rheumatoid arthritis
C) Fibromyalgia
D) Gout

Rationale: The patient’s chronic muscle pain, tenderness, sleep disturbances, and fatigue are indicative of fibromyalgia, a musculoskeletal disorder characterized by widespread pain.

HESI Exit Question 49: A 50-year-old male presents with acute lower back pain and reports a recent history of heavy lifting. On physical examination, you note muscle spasm and limited range of motion in the lumbar spine. What condition is most likely causing these symptoms?
A) Herniated disc (lumbar disc herniation)
B) Osteoarthritis
C) Ankylosing spondylitis
D) Gout

Rationale: The patient’s acute lower back pain, history of heavy lifting, muscle spasm, and limited range of motion are suggestive of a herniated disc (lumbar disc herniation).

HESI Exit Question 50: A 65-year-old female with a history of osteoporosis presents with a fracture of her wrist after a minor fall. What medication is commonly prescribed to manage osteoporosis and reduce the risk of fractures?
A) Allopurinol (Zyloprim)
B) Methotrexate (Rheumatrex)
C) Alendronate (Fosamax)
D) Prednisone

Rationale: Alendronate is a bisphosphonate medication commonly prescribed to manage osteoporosis and reduce the risk of fractures by increasing bone density.

rHEUMATORID ARTHRITIS, NCLEX, ANCC, AANP, HESI EXIT, HESI A2
Joint damage in rheumatoid arthritis

HESI Exit Question 51: A 40-year-old male with a history of rheumatoid arthritis presents with joint pain and inflammation that has not responded to nonsteroidal anti-inflammatory drugs (NSAIDs). What type of medication is often used as a disease-modifying antirheumatic drug (DMARD) in this patient?
A) Prednisone
B) Ibuprofen (Advil)
C) Methotrexate (Rheumatrex)
D) Allopurinol (Zyloprim)

Rationale: Methotrexate is a commonly used disease-modifying antirheumatic drug (DMARD) in the treatment of rheumatoid arthritis, as it helps to slow the progression of the disease and reduce inflammation.

HESI Exit Question 52: A 55-year-old female with a history of gout presents with recurrent gout attacks. What medication is commonly used to prevent gout attacks by lowering uric acid levels?
A) Prednisone
B) Allopurinol (Zyloprim)
C) Methotrexate (Rheumatrex)
D) Alendronate (Fosamax)

Rationale: Allopurinol is a medication commonly used to prevent gout attacks by lowering uric acid levels in the blood.

HESI Exit Question 53: A 50-year-old male with a history of ankylosing spondylitis experiences chronic pain and stiffness in the spine. What type of medication is often used to manage pain and inflammation in this condition?
A) Prednisone
B) Ibuprofen (Advil)
C) Methotrexate (Rheumatrex)
D) Allopurinol (Zyloprim)

Rationale: Nonsteroidal anti-inflammatory drugs (NSAIDs) like ibuprofen are commonly used to manage pain and inflammation in ankylosing spondylitis.

HESI Exit Question 54: A 60-year-old female with a history of osteoarthritis presents with moderate knee pain and difficulty with daily activities. What type of medication is often recommended for symptomatic relief in osteoarthritis?
A) Prednisone
B) Ibuprofen (Advil)
C) Methotrexate (Rheumatrex)
D) Hyaluronic acid injections

Rationale: Hyaluronic acid injections are often recommended for symptomatic relief in osteoarthritis, particularly in knee joints, to improve joint lubrication and reduce pain.

HESI Exit Question 55: A 45-year-old male with a history of osteoarthritis presents with knee pain that is not adequately controlled with over-the-counter pain relievers. What medication class is often prescribed for osteoarthritis pain management when nonprescription drugs are insufficient?
A) Nonsteroidal anti-inflammatory drugs (NSAIDs)
B) Disease-modifying antirheumatic drugs (DMARDs)
C) Bisphosphonates
D) Anticholinergics

Rationale: Nonsteroidal anti-inflammatory drugs (NSAIDs) are commonly prescribed for pain management in osteoarthritis when nonprescription drugs are insufficient to control pain and inflammation.

HESI Exit Question 56: A 70-year-old female with a history of osteoporosis presents with a vertebral compression fracture. What medication class is often prescribed to reduce the risk of future fractures in patients with osteoporosis?
A) Nonsteroidal anti-inflammatory drugs (NSAIDs)
B) Disease-modifying antirheumatic drugs (DMARDs)
C) Bisphosphonates
D) Anticholinergics

Rationale: Bisphosphonates are commonly prescribed to reduce the risk of future fractures in patients with osteoporosis by increasing bone density.

HESI Exit Question 57: A 35-year-old male with a history of osteoarthritis experiences joint pain and inflammation. He prefers a topical medication for pain relief. What type of topical medication is commonly used for osteoarthritis pain management?
A) Lidocaine patch
B) Hyaluronic acid gel
C) Prednisone cream
D) Methotrexate ointment

Rationale: Lidocaine patches are commonly used as a topical option for pain relief in osteoarthritis by providing local anesthesia to numb the area and alleviate pain.

HESI Exit Question 58: A 65-year-old male presents with a fracture of his hip after a fall. What medication class is often used to manage osteoporosis and reduce the risk of fractures in older adults?
A) Nonsteroidal anti-inflammatory drugs (NSAIDs)
B) Disease-modifying antirheumatic drugs (DMARDs)
C) Bisphosphonates
D) Anticholinergics

Rationale: Bisphosphonates are commonly used to manage osteoporosis and reduce the risk of fractures, particularly in older adults.

HESI Exit Question 59: A 40-year-old female with a history of rheumatoid arthritis experiences joint pain and inflammation. She prefers a topical medication for pain relief. What type of topical medication is commonly used for rheumatoid arthritis pain management?
A) Lidocaine patch
B) Hyaluronic acid gel
C) Prednisone cream
D) Methotrexate ointment

Rationale: Methotrexate ointment is not commonly used for rheumatoid arthritis pain management. Methotrexate is typically administered as an oral or injectable medication to treat the underlying disease.

HESI Exit Question 60: A 55-year-old male with a history of gout presents with severe joint pain, redness, and swelling in his big toe. What medication class is often prescribed to provide rapid pain relief during acute gout attacks?
A) Nonsteroidal anti-inflammatory drugs (NSAIDs)
B) Disease-modifying antirheumatic drugs (DMARDs)
C) Bisphosphonates
D) Anticholinergics

Rationale: Nonsteroidal anti-inflammatory drugs (NSAIDs) are often prescribed to provide rapid pain relief during acute gout attacks by reducing inflammation and pain.

Gout of the great toe. NCLEX, ANCC, AANP, HESI EXIT.
Gout typically affects the big toe.

HESI Exit Question 61: A 45-year-old male presents with recurrent episodes of burning chest pain that worsens after meals and when lying down. He also experiences regurgitation of stomach contents into the throat. What condition is most likely causing these symptoms?
A) Gastric ulcer
B) Gastroesophageal reflux disease (GERD)
C) Esophageal cancer
D) Peptic ulcer

Rationale: The patient’s symptoms of burning chest pain, worsened after meals and when lying down, along with regurgitation, are indicative of GERD, a condition where stomach acid flows back into the esophagus.

HESI Exit Question 62: A 60-year-old female presents with chronic abdominal pain, bloating, and changes in bowel habits, including diarrhea and constipation. She also reports a family history of colon cancer. What condition should be suspected in this patient?
A) Gastric ulcer
B) Gastroesophageal reflux disease (GERD)
C) Irritable bowel syndrome (IBS)
D) Peptic ulcer

Rationale: The patient’s symptoms of chronic abdominal pain, bloating, and changes in bowel habits, along with a family history of colon cancer, are suggestive of IBS, a functional gastrointestinal disorder.

Read more about Crohn’s disease vs ulcerative colitis

HESI Exit Question 63: A 35-year-old male presents with epigastric pain that is relieved with food intake. He also reports a history of regular aspirin use. What condition is most likely causing these symptoms?
A) Gastric ulcer
B) Gastroesophageal reflux disease (GERD)
C) Esophageal cancer
D) Peptic ulcer

Rationale: The patient’s epigastric pain relieved with food intake, along with a history of aspirin use, is suggestive of a peptic ulcer, which can be caused or exacerbated by aspirin and other nonsteroidal anti-inflammatory drugs (NSAIDs).

HESI Exit Question 64: A 55-year-old female presents with recurrent episodes of abdominal pain, bloating, and excessive burping. She also reports feeling full quickly during meals. What condition should be suspected in this patient?
A) Gastric ulcer
B) Gastroesophageal reflux disease (GERD)
C) Peptic ulcer
D) Gastroparesis

Rationale: The patient’s symptoms of abdominal pain, bloating, excessive burping, and feeling full quickly during meals are indicative of gastroparesis, a condition where the stomach takes longer to empty its contents.

HESI Exit Question 65: A 50-year-old male presents with severe, burning epigastric pain that often occurs at night and is relieved by antacids. He also reports a history of smoking and alcohol consumption. What condition is most likely causing these symptoms?
A) Gastric ulcer
B) Gastroesophageal reflux disease (GERD)
C) Esophageal cancer
D) Peptic ulcer

Rationale: The patient’s severe, burning epigastric pain relieved by antacids, along with a history of smoking and alcohol consumption, is suggestive of a gastric ulcer.

HESI Exit Question 66: A 65-year-old male presents with difficulty swallowing, especially with solid foods. He has unintentionally lost weight and reports a history of tobacco use. What condition should be suspected in this patient?
A) Gastric ulcer
B) Gastroesophageal reflux disease (GERD)
C) Esophageal cancer
D) Peptic ulcer

Rationale: The patient’s difficulty swallowing, unintentional weight loss, and history of tobacco use raise concern for esophageal cancer, which can obstruct the passage of food.

HESI Exit Question 67: A 45-year-old male presents with recurrent episodes of abdominal pain, particularly after meals. He reports a history of excessive alcohol consumption. What condition is most likely causing these symptoms?
A) Gastric ulcer
B) Gastroesophageal reflux disease (GERD)
C) Alcoholic gastritis
D) Peptic ulcer

Rationale: The patient’s recurrent abdominal pain, particularly after meals, along with a history of excessive alcohol consumption, is suggestive of alcoholic gastritis, an inflammation of the stomach lining due to alcohol abuse.

HESI Exit Question 68: A 60-year-old female presents with dark, tarry stools and fatigue. She also reports a history of chronic NSAID use for pain management. What condition is most likely causing these symptoms?
A) Gastric ulcer
B) Gastroesophageal reflux disease (GERD)
C) Esophageal cancer
D) Gastrointestinal bleeding

Rationale: The patient’s dark, tarry stools and fatigue, along with a history of chronic NSAID use, are indicative of gastrointestinal bleeding, which can occur as a result of NSAID-induced gastric irritation.

HESI Exit Question 69: A 35-year-old male presents with chronic, burning epigastric pain that improves with food intake but recurs after meals. He also reports a history of Helicobacter pylori infection. What condition is most likely causing these symptoms?
A) Gastric ulcer
B) Gastroesophageal reflux disease (GERD)
C) Esophageal cancer
D) Peptic ulcer

Rationale: The patient’s chronic epigastric pain that improves with food intake but recurs after meals, along with a history of Helicobacter pylori infection, is suggestive of a peptic ulcer.

HESI Exit Question 70: A 55-year-old female presents with recurrent episodes of regurgitation of undigested food and difficulty swallowing solid foods. She also reports a history of obesity. What condition should be suspected in this patient?
A) Gastric ulcer
B) Gastroesophageal reflux disease (GERD)
C) Esophageal cancer
D) Gastroesophageal reflux (GERD)

Rationale: The patient’s symptoms of regurgitation of undigested food and difficulty swallowing solid foods, along with a history of obesity, are indicative of GERD, a condition where stomach contents flow back into the esophagus.

HESI Exit Question 71: A 50-year-old male presents with recurrent, burning chest pain that worsens with spicy foods. He also reports a family history of gastric cancer. What condition is most likely causing these symptoms?
A) Gastric ulcer
B) Gastroesophageal reflux disease (GERD)
C) Esophageal cancer
D) Peptic ulcer

Rationale: The patient’s recurrent, burning chest pain worsened by spicy foods, along with a family history of gastric cancer, is suggestive of a gastric ulcer.

HESI Exit Question 72: A 65-year-old male presents with persistent heartburn and regurgitation of stomach contents into the throat, especially after meals. What condition is most likely causing these symptoms?
A) Gastric ulcer
B) Gastroesophageal reflux disease (GERD)
C) Esophageal cancer
D) Peptic ulcer

Rationale: The patient’s persistent heartburn and regurgitation of stomach contents, especially after meals, are indicative of GERD, a condition where stomach acid flows back into the esophagus.

HESI Exit Question 73: A 40-year-old male presents with a sudden, severe upper abdominal pain radiating to the back. He reports a history of heavy alcohol consumption. What condition is most likely causing these symptoms?
A) Gastric ulcer
B) Gastroesophageal reflux disease (GERD)
C) Acute pancreatitis
D) Peptic ulcer

Rationale: The patient’s sudden, severe upper abdominal pain radiating to the back, along with a history of heavy alcohol consumption, is suggestive of acute pancreatitis.

Do more endocrine questions

HESI Exit Question 74: A 55-year-old female presents with chronic, gnawing epigastric pain that is relieved by food intake. She also reports a history of chronic NSAID use for arthritis. What condition should be suspected in this patient?
A) Gastric ulcer
B) Gastroesophageal reflux disease (GERD)
C) Chronic gastritis
D) Peptic ulcer

Rationale: The patient’s chronic, gnawing epigastric pain relieved by food intake, along with a history of chronic NSAID use, is suggestive of a peptic ulcer.

Gastroesophageal reflux (GERD). NCLEX, ANCC, AANP, HESI EXIT, HESI A2
Gastroesophageal reflux (GERD)

HESI Exit Question 75: A 60-year-old female presents with chronic, upper abdominal pain, bloating, and early satiety. She also reports unintentional weight loss. What condition should be suspected in this patient?
A) Gastric ulcer
B) Gastroesophageal reflux disease (GERD)
C) Gastric cancer
D) Peptic ulcer

Rationale: The patient’s chronic upper abdominal pain, bloating, early satiety, and unintentional weight loss raise concern for gastric cancer, which can cause obstructive symptoms.

HESI Exit Question 76: A 40-year-old male presents with chronic diarrhea, abdominal pain, and weight loss. He also reports a family history of inflammatory bowel disease (IBD). What condition should be suspected in this patient?
A) Irritable bowel syndrome (IBS)
B) Diverticulitis
C) Crohn’s disease
D) Celiac disease

Rationale: The patient’s symptoms of chronic diarrhea, abdominal pain, weight loss, and a family history of IBD are suggestive of Crohn’s disease, a type of IBD.

HESI Exit Question 77: A 50-year-old female presents with lower abdominal pain, fever, and leukocytosis. She reports a history of diverticulosis. What condition is most likely causing these symptoms?
A) Irritable bowel syndrome (IBS)
B) Diverticulitis
C) Crohn’s disease
D) Celiac disease

Rationale: The patient’s lower abdominal pain, fever, leukocytosis, and history of diverticulosis are indicative of diverticulitis, an inflammatory condition of the diverticula.

HESI Exit Question 78: A 30-year-old male presents with chronic diarrhea, abdominal cramping, and bloating. He also reports a history of lactose intolerance. What condition should be suspected in this patient?
A) Irritable bowel syndrome (IBS)
B) Diverticulitis
C) Crohn’s disease
D) Celiac disease

Rationale: The patient’s symptoms of chronic diarrhea, abdominal cramping, bloating, and a history of lactose intolerance are indicative of IBS.

HESI Exit Question 79: A 45-year-old male presents with bright red blood in his stools and fatigue. He also reports a family history of colorectal cancer. What condition should be suspected in this patient?
A) Irritable bowel syndrome (IBS)
B) Diverticulitis
C) Colorectal cancer
D) Celiac disease

Rationale: The patient’s symptoms of bright red blood in stools, fatigue, and a family history of colorectal cancer raise concern for colorectal cancer, which can present with gastrointestinal bleeding.

HESI Exit Question 80: A 35-year-old female presents with chronic diarrhea, bloating, and unintentional weight loss. She also reports a history of autoimmune disorders. What condition is most likely causing these symptoms?
A) Irritable bowel syndrome (IBS)
B) Diverticulitis
C) Crohn’s disease
D) Celiac disease

Rationale: The patient’s chronic diarrhea, bloating, unintentional weight loss, and a history of autoimmune disorders are suggestive of celiac disease, an autoimmune disorder triggered by gluten ingestion.

HESI Exit Question 81: A 60-year-old male presents with chronic constipation, abdominal pain, and a feeling of incomplete evacuation. He also reports a family history of colon cancer. What condition should be suspected in this patient?
A) Irritable bowel syndrome (IBS)
B) Diverticulitis
C) Colorectal cancer
D) Celiac disease

Rationale: The patient’s symptoms of chronic constipation, abdominal pain, feeling of incomplete evacuation, and a family history of colon cancer are indicative of IBS.

HESI Exit Question 82: A 50-year-old male presents with chronic diarrhea, abdominal pain, and rectal bleeding. He reports a history of excessive alcohol consumption. What condition is most likely causing these symptoms?
A) Irritable bowel syndrome (IBS)
B) Diverticulitis
C) Colorectal cancer
D) Alcohol-induced gastritis

Rationale: The patient’s chronic diarrhea, abdominal pain, rectal bleeding, and a history of excessive alcohol consumption raise concern for colorectal cancer, which can present with gastrointestinal bleeding.

HESI Exit Question 83: A 45-year-old female presents with chronic diarrhea, abdominal cramping, and fatigue. She also reports a history of diverticulosis. What condition should be suspected in this patient?
A) Irritable bowel syndrome (IBS)
B) Diverticulitis
C) Crohn’s disease
D) Celiac disease

Rationale: The patient’s symptoms of chronic diarrhea, abdominal cramping, fatigue, and a history of diverticulosis are indicative of IBS.

HESI Exit Question 84: A 40-year-old male presents with chronic diarrhea, abdominal pain, and unintentional weight loss. He also reports a family history of celiac disease. What condition should be suspected in this patient?
A) Irritable bowel syndrome (IBS)
B) Diverticulitis
C) Celiac disease
D) Inflammatory bowel disease (IBD)

Rationale: The patient’s chronic diarrhea, abdominal pain, unintentional weight loss, and family history of celiac disease raise concern for IBD, which includes Crohn’s disease and ulcerative colitis.

HESI Exit Question 85: A 55-year-old female presents with chronic constipation, abdominal pain, and bloating. She also reports a history of multiple abdominal surgeries. What condition is most likely causing these symptoms?
A) Irritable bowel syndrome (IBS)
B) Diverticulitis
C) Colorectal cancer
D) Adhesive small bowel obstruction

Rationale: The patient’s chronic constipation, abdominal pain, bloating, and history of multiple abdominal surgeries suggest an adhesive small bowel obstruction, a complication of abdominal surgeries.

HESI Exit Question 86: A 30-year-old male presents with chronic diarrhea, abdominal pain, and joint pain. He also reports a family history of autoimmune disorders. What condition should be suspected in this patient?
A) Irritable bowel syndrome (IBS)
B) Diverticulitis
C) Inflammatory bowel disease (IBD)
D) Celiac disease

Rationale: The patient’s chronic diarrhea, abdominal pain, joint pain, family history of autoimmune disorders, and age raise concern for IBD, which includes Crohn’s disease and ulcerative colitis.

HESI Exit Question 87: A 60-year-old male presents with chronic diarrhea, abdominal cramping, and weight loss. He also reports a history of smoking. What condition should be suspected in this patient?
A) Irritable bowel syndrome (IBS)
B) Diverticulitis
C) Colorectal cancer
D) Chronic gastritis

Rationale: The patient’s chronic diarrhea, abdominal cramping, weight loss, and history of smoking raise concern for colorectal cancer, which can present with these symptoms.

HESI Exit Question 88: A 35-year-old female presents with chronic constipation, abdominal pain, and bloating. She also reports a history of abdominal surgeries for endometriosis. What condition is most likely causing these symptoms?
A) Irritable bowel syndrome (IBS)
B) Diverticulitis
C) Adhesive small bowel obstruction
D) Celiac disease

Rationale: The patient’s chronic constipation, abdominal pain, bloating, and history of abdominal surgeries for endometriosis suggest an adhesive small bowel obstruction, a complication of surgeries.

HESI Exit Question 89: A 50-year-old male presents with bright red blood in his stools and fatigue. He also reports a history of alcohol abuse. What condition should be suspected in this patient?
A) Irritable bowel syndrome (IBS)
B) Diverticulitis
C) Colorectal cancer
D) Alcoholic gastritis

Rationale: The patient’s symptoms of bright red blood in stools, fatigue, and a history of alcohol abuse raise concern for colorectal cancer, which can present with gastrointestinal bleeding.

HESI Exit Question 90: A 45-year-old female presents with chronic diarrhea, abdominal cramping, and joint pain. She also reports a family history of celiac disease. What condition should be suspected in this patient?
A) Irritable bowel syndrome (IBS)
B) Diverticulitis
C) Inflammatory bowel disease (IBD)
D) Celiac disease

Rationale: The patient’s chronic diarrhea, abdominal cramping, joint pain, family history of celiac disease, and age raise concern for celiac disease, an autoimmune disorder triggered by gluten ingestion.

HESI Exit Question 91: A 45-year-old female presents with severe right upper quadrant abdominal pain that radiates to her right shoulder. She also reports nausea and vomiting after eating fatty foods. What condition should be suspected in this patient?
A) Cholecystitis
B) Pancreatitis
C) Gastric ulcer
D) Inflammatory bowel disease (IBD)

Rationale: The patient’s severe right upper quadrant abdominal pain, radiating to the right shoulder, along with nausea and vomiting after eating fatty foods, is indicative of cholecystitis, inflammation of the gallbladder.

HESI Exit Question 92: A 60-year-old male presents with painless jaundice, pale stools, and dark urine. He also reports weight loss. What condition is most likely causing these symptoms?
A) Cholecystitis
B) Pancreatitis
C) Gallstones
D) Biliary obstruction

Rationale: The patient’s painless jaundice, pale stools, dark urine, and weight loss are suggestive of biliary obstruction, often caused by gallstones or other obstructions in the bile ducts.

HESI Exit Question 93: A 35-year-old female presents with recurrent episodes of right upper quadrant abdominal pain that occur after eating greasy or fried foods. She also reports bloating and gas. What condition should be suspected in this patient?
A) Cholecystitis
B) Pancreatitis
C) Gastric ulcer
D) Diverticulitis

Rationale: The patient’s recurrent right upper quadrant abdominal pain after eating greasy or fried foods, along with bloating and gas, is indicative of cholecystitis, often triggered by gallstones.

HESI Exit Question 94: A 50-year-old male presents with severe right upper quadrant abdominal pain, fever, and elevated white blood cell count. He also reports a history of gallstones. What condition is most likely causing these symptoms?
A) Cholecystitis
B) Pancreatitis
C) Gastric ulcer
D) Irritable bowel syndrome (IBS)

Rationale: The patient’s severe right upper quadrant abdominal pain, fever, elevated white blood cell count, and history of gallstones are indicative of cholecystitis, inflammation of the gallbladder.

HESI Exit Question 95: A 40-year-old female presents with upper abdominal pain, bloating, and nausea. She also reports intolerance to fatty foods. What condition should be suspected in this patient?
A) Cholecystitis
B) Pancreatitis
C) Gastric ulcer
D) Celiac disease

Rationale: The patient’s upper abdominal pain, bloating, nausea, and intolerance to fatty foods are suggestive of cholecystitis, often triggered by gallstones.

HESI Exit Question 96: A 55-year-old female presents with recurrent right upper quadrant abdominal pain that occurs several hours after eating a heavy meal. She also reports a history of gallstones. What condition is most likely causing these symptoms?
A) Cholecystitis
B) Pancreatitis
C) Gastric ulcer
D) Gastroesophageal reflux disease (GERD)

Rationale: The patient’s recurrent right upper quadrant abdominal pain occurring several hours after eating a heavy meal, along with a history of gallstones, is indicative of cholecystitis, inflammation of the gallbladder.

HESI Exit Question 97: A 65-year-old male presents with right upper quadrant abdominal pain and tenderness, as well as jaundice. He also reports clay-colored stools and dark urine. What condition should be suspected in this patient?
A) Cholecystitis
B) Pancreatitis
C) Gallstones
D) Biliary obstruction

Rationale: The patient’s right upper quadrant abdominal pain and tenderness, jaundice, clay-colored stools, and dark urine are suggestive of biliary obstruction, often caused by gallstones or other obstructions in the bile ducts.

HESI Exit Question 98: A 35-year-old female presents with recurrent episodes of right upper quadrant abdominal pain, especially after consuming fatty meals. She also reports shoulder pain. What condition is most likely causing these symptoms?
A) Cholecystitis
B) Pancreatitis
C) Gastric ulcer
D) Inflammatory bowel disease (IBD)

Rationale: The patient’s recurrent right upper quadrant abdominal pain, worsened after consuming fatty meals, and shoulder pain are indicative of cholecystitis, often triggered by gallstones.

HESI Exit Question 99: A 50-year-old male presents with severe right upper quadrant abdominal pain that radiates to his back. He also reports nausea and vomiting. What condition should be suspected in this patient?
A) Cholecystitis
B) Pancreatitis
C) Gallstones
D) Biliary obstruction

Rationale: The patient’s severe right upper quadrant abdominal pain radiating to his back, along with nausea and vomiting, is indicative of pancreatitis, inflammation of the pancreas, often related to gallstones or alcohol consumption.

HESI Exit Question 100: A 45-year-old female presents with upper abdominal pain, bloating, and intolerance to fatty foods. She also reports a family history of gallbladder disease. What condition is most likely causing these symptoms?
A) Cholecystitis
B) Pancreatitis
C) Gallstones
D) Inflammatory bowel disease (IBD)

Rationale: The patient’s upper abdominal pain, bloating, intolerance to fatty foods, and family history of gallbladder disease suggest the presence of gallstones, which can lead to gallbladder-related symptoms.

HESI Exit Question 101: A 40-year-old male presents with right upper quadrant abdominal pain and tenderness. He also reports dark urine and pale stools. What condition should be suspected in this patient?
A) Cholecystitis
B) Pancreatitis
C) Gallstones
D) Biliary obstruction

Rationale: The patient’s right upper quadrant abdominal pain and tenderness, along with dark urine and pale stools, are suggestive of biliary obstruction, often caused by gallstones or other obstructions in the bile ducts.

HESI Exit Question 102: A 55-year-old female presents with severe right upper quadrant abdominal pain that radiates to her back. She also reports fever and elevated white blood cell count. What condition is most likely causing these symptoms?
A) Cholecystitis
B) Pancreatitis
C) Gallstones
D) Gastroesophageal reflux disease (GERD)

Rationale: The patient’s severe right upper quadrant abdominal pain radiating to her back, fever, and elevated white blood cell count are indicative of pancreatitis, often related to gallstones or alcohol consumption.

HESI Exit Question 103: A 60-year-old male presents with recurrent right upper quadrant abdominal pain, especially after consuming fatty meals. He also reports flatulence and diarrhea. What condition should be suspected in this patient?
A) Cholecystitis
B) Pancreatitis
C) Gallstones
D) Celiac disease

Rationale: The patient’s recurrent right upper quadrant abdominal pain after consuming fatty meals, flatulence, and diarrhea are suggestive of gallstones, which can lead to gastrointestinal symptoms.

HESI Exit Question 104: A 35-year-old female presents with right upper quadrant abdominal pain and tenderness, as well as jaundice. She also reports dark urine and pale stools. What condition is most likely causing these symptoms?
A) Cholecystitis
B) Pancreatitis
C) Gallstones
D) Biliary obstruction

Rationale: The patient’s right upper quadrant abdominal pain and tenderness, jaundice, dark urine, and pale stools are indicative of biliary obstruction, often caused by gallstones or other obstructions in the bile ducts.

HESI Exit Question 105: A 35-year-old male presents with easy bruising, petechiae, and fatigue. He also reports recurrent infections. Physical examination reveals splenomegaly. What condition should be suspected in this patient?
A) Sickle cell disease
B) Leukemia
C) Thrombocytopenia
D) Infectious mononucleosis

Rationale: The patient’s easy bruising, petechiae, fatigue, recurrent infections, and splenomegaly are indicative of thrombocytopenia, a condition characterized by a low platelet count.

HESI Exit Question 106: A 40-year-old female presents with left upper quadrant abdominal pain and fullness. She also reports early satiety and unintentional weight loss. What condition is most likely causing these symptoms?
A) Sickle cell disease
B) Leukemia
C) Splenomegaly
D) Autoimmune hemolytic anemia

Rationale: The patient’s left upper quadrant abdominal pain, fullness, early satiety, and unintentional weight loss are suggestive of splenomegaly, an enlarged spleen.

HESI Exit Question 107: A 50-year-old male presents with severe left upper quadrant abdominal pain that radiates to his left shoulder. He also reports dizziness and pale skin. What condition should be suspected in this patient?
A) Sickle cell disease
B) Leukemia
C) Splenic rupture
D) Rheumatoid arthritis

Rationale: The patient’s severe left upper quadrant abdominal pain, radiating to the left shoulder, dizziness, and pale skin are indicative of splenic rupture, a medical emergency often associated with trauma.

HESI Exit Question 108: A 45-year-old female presents with fatigue, joint pain, and butterfly-shaped rash on her face. She also reports a family history of autoimmune diseases. What condition is most likely causing these symptoms?
A) Sickle cell disease
B) Leukemia
C) Lupus (Systemic lupus erythematosus)
D) Thrombocytopenia

Rationale: The patient’s fatigue, joint pain, butterfly-shaped rash on her face, and family history of autoimmune diseases are suggestive of lupus (systemic lupus erythematosus), an autoimmune disorder.

HESI Exit Question 109: A 55-year-old male presents with recurrent bacterial infections, particularly sinus and lung infections. He also reports a history of splenectomy. What condition should be suspected in this patient?
A) Sickle cell disease
B) Leukemia
C) Splenic sequestration
D) Functional asplenia

Rationale: The patient’s recurrent bacterial infections, history of splenectomy, and absence of a functioning spleen suggest functional asplenia, where the spleen’s immune functions are compromised.

HESI Exit Question 110: A 30-year-old female presents with abdominal pain and discomfort in the left upper quadrant. She also reports a family history of lymphoma. What condition is most likely causing these symptoms?
A) Sickle cell disease
B) Leukemia
C) Splenomegaly
D) Hodgkin lymphoma

Rationale: The patient’s left upper quadrant abdominal pain, family history of lymphoma, and lymphadenopathy are suggestive of Hodgkin lymphoma, which can involve the spleen.

HESI Exit Question 111: A 60-year-old male presents with recurrent infections, night sweats, and unexplained weight loss. Physical examination reveals splenomegaly. What condition should be suspected in this patient?
A) Sickle cell disease
B) Leukemia
C) Thrombocytopenia
D) Non-Hodgkin lymphoma

Rationale: The patient’s recurrent infections, night sweats, unexplained weight loss, splenomegaly, and age are suggestive of non-Hodgkin lymphoma, which can involve the spleen.

HESI Exit Question 112: A 40-year-old female presents with recurrent oral ulcers, joint pain, and photosensitivity. She also reports a family history of autoimmune diseases. What condition is most likely causing these symptoms?
A) Sickle cell disease
B) Leukemia
C) Systemic sclerosis (Scleroderma)
D) Systemic lupus erythematosus (SLE)

Rationale: The patient’s recurrent oral ulcers, joint pain, photosensitivity, family history of autoimmune diseases, and the age of onset are suggestive of systemic lupus erythematosus (SLE), an autoimmune disorder.

HESI Exit Question 113: A 50-year-old male presents with recurrent infections, particularly skin and respiratory tract infections. He also reports a history of splenectomy. What condition should be suspected in this patient?
A) Sickle cell disease
B) Leukemia
C) Functional asplenia
D) Hemophilia

Rationale: The patient’s recurrent infections, history of splenectomy, and absence of a functioning spleen suggest functional asplenia, where the spleen’s immune functions are compromised.

HESI Exit Question 114: A 45-year-old female presents with joint pain, dry eyes, and dry mouth. She also reports difficulty swallowing and hoarseness. What condition is most likely causing these symptoms?
A) Sickle cell disease
B) Leukemia
C) Sjögren’s syndrome
D) Rheumatoid arthritis

Rationale: The patient’s joint pain, dry eyes, dry mouth, difficulty swallowing, hoarseness, and age are suggestive of Sjögren’s syndrome, an autoimmune disorder.

HESI Exit Question 115: A 55-year-old male presents with abdominal pain, fullness, and discomfort in the left upper quadrant. He also reports a history of chronic liver disease. What condition should be suspected in this patient?
A) Sickle cell disease
B) Leukemia
C) Splenomegaly
D) Portal hypertension

Rationale: The patient’s abdominal pain, fullness, discomfort in the left upper quadrant, history of chronic liver disease, and splenomegaly suggest splenomegaly due to portal hypertension.

HESI Exit Question 116: A 35-year-old female presents with abdominal pain, fatigue, and pale skin. She also reports heavy menstrual bleeding. What condition is most likely causing these symptoms?
A) Sickle cell disease
B) Leukemia
C) Hemolytic anemia
D) Hemophilia

Rationale: The patient’s abdominal pain, fatigue, pale skin, heavy menstrual bleeding, and signs of hemolysis are indicative of hemolytic anemia, a condition where red blood cells are destroyed prematurely.

HESI Exit Question 117: A 60-year-old male presents with recurrent infections, particularly pneumonia and sinusitis. He also reports splenomegaly and a history of alcohol abuse. What condition should be suspected in this patient?
A) Sickle cell disease
B) Leukemia
C) Functional asplenia
D) Cirrhosis

Rationale: The patient’s recurrent infections, splenomegaly, history of alcohol abuse, and signs of liver disease are suggestive of cirrhosis, which can lead to immune dysfunction and splenomegaly.

HESI Exit Question 118: A 30-year-old female presents with abdominal pain, joint pain, and a butterfly-shaped rash on her face. She also reports a family history of autoimmune diseases. What condition is most likely causing these symptoms?
A) Sickle cell disease
B) Leukemia
C) Systemic sclerosis (Scleroderma)
D) Systemic lupus erythematosus (SLE)

Rationale: The patient’s abdominal pain, joint pain, butterfly-shaped rash on her face, family history of autoimmune diseases, and age are suggestive of systemic lupus erythematosus (SLE), an autoimmune disorder.

HESI Exit Question 119: A 40-year-old male presents with abdominal pain, discomfort in the left upper quadrant, and early satiety. He also reports a history of autoimmune disorders. What condition should be suspected in this patient?
A) Sickle cell disease
B) Leukemia
C) Splenomegaly
D) Systemic sclerosis (Scleroderma)

Rationale: The patient’s abdominal pain, discomfort in the left upper quadrant, early satiety, history of autoimmune disorders, and age are suggestive of systemic sclerosis (scleroderma), an autoimmune disorder.

HESI Exit Question 120: A 50-year-old female presents with fatigue, joint pain, and morning stiffness. She also reports a family history of autoimmune diseases. What condition is most likely causing these symptoms?
A) Sickle cell disease
B) Leukemia
C) Rheumatoid arthritis
D) Hemophilia

Rationale: The patient’s fatigue, joint pain, morning stiffness, family history of autoimmune diseases, and age are suggestive of rheumatoid arthritis, an autoimmune disorder that primarily affects the joints.

HESI Exit Question 121: A 45-year-old male presents with severe abdominal pain that radiates to his back. He also reports nausea and vomiting. Laboratory tests reveal elevated amylase and lipase levels. What condition should be suspected in this patient?
A) Gastric ulcer
B) Pancreatitis
C) Gastroesophageal reflux disease (GERD)
D) Inflammatory bowel disease (IBD)

Rationale: The patient’s severe abdominal pain radiating to his back, nausea, vomiting, and elevated amylase and lipase levels are indicative of pancreatitis, inflammation of the pancreas.

HESI Exit Question 122: A 55-year-old female presents with unexplained weight loss, pale stools, and dark urine. She also reports abdominal pain that worsens after eating fatty foods. What condition is most likely causing these symptoms?
A) Gastric ulcer
B) Pancreatitis
C) Cholecystitis
D) Gallstones

Rationale: The patient’s unexplained weight loss, pale stools, dark urine, and abdominal pain that worsens after eating fatty foods are suggestive of pancreatitis, inflammation of the pancreas.

HESI Exit Question 123: A 40-year-old male presents with abdominal pain, bloating, and diarrhea after consuming fatty meals. He also reports a history of alcohol abuse. What condition should be suspected in this patient?
A) Gastric ulcer
B) Pancreatitis
C) Gastroesophageal reflux disease (GERD)
D) Celiac disease

Rationale: The patient’s abdominal pain, bloating, diarrhea after consuming fatty meals, and history of alcohol abuse raise concern for pancreatitis, inflammation of the pancreas.

HESI Exit Question 124: A 50-year-old female presents with jaundice, abdominal pain, and pale stools. She also reports dark urine and unexplained weight loss. What condition is most likely causing these symptoms?
A) Gastric ulcer
B) Pancreatitis
C) Cholecystitis
D) Pancreatic cancer

Rationale: The patient’s jaundice, abdominal pain, pale stools, dark urine, and unexplained weight loss are suggestive of pancreatic cancer, which can cause obstructive jaundice and digestive symptoms.

HESI Exit Question 125: A 35-year-old female presents with abdominal pain, nausea, and vomiting. She also reports a history of alcohol abuse. What condition should be suspected in this patient?
A) Gastric ulcer
B) Pancreatitis
C) Gastroesophageal reflux disease (GERD)
D) Cholecystitis

Rationale: The patient’s abdominal pain, nausea, vomiting, and history of alcohol abuse raise concern for pancreatitis, inflammation of the pancreas.

HESI Exit Question 126: A 60-year-old male presents with unexplained weight loss, abdominal pain, and diarrhea. He also reports a family history of pancreatic cancer. What condition is most likely causing these symptoms?
A) Gastric ulcer
B) Pancreatitis
C) Cholecystitis
D) Pancreatic cancer

Rationale: The patient’s unexplained weight loss, abdominal pain, diarrhea, family history of pancreatic cancer, and age are suggestive of pancreatic cancer, which can present with these symptoms.

HESI Exit Question 127: A 45-year-old female presents with upper abdominal pain, bloating, and a feeling of fullness. She also reports unexplained weight loss. What condition should be suspected in this patient?
A) Gastric ulcer
B) Pancreatitis
C) Gastroesophageal reflux disease (GERD)
D) Pancreatic cancer

Rationale: The patient’s upper abdominal pain, bloating, feeling of fullness, unexplained weight loss, and age are suggestive of pancreatic cancer, which can present with digestive symptoms.

HESI Exit Question 128: A 55-year-old male presents with abdominal pain that radiates to his back. He also reports jaundice and pale stools. What condition is most likely causing these symptoms?
A) Gastric ulcer
B) Pancreatitis
C) Cholecystitis
D) Pancreatic cancer

Rationale: The patient’s abdominal pain radiating to his back, jaundice, pale stools, and age are suggestive of pancreatic cancer, which can cause obstructive jaundice.

HESI Exit Question 129: A 30-year-old female presents with upper abdominal pain, bloating, and diarrhea after consuming fatty meals. She also reports a family history of pancreatic cancer. What condition should be suspected in this patient?
A) Gastric ulcer
B) Pancreatitis
C) Gastroesophageal reflux disease (GERD)
D) Chronic pancreatitis

Rationale: The patient’s upper abdominal pain, bloating, diarrhea after consuming fatty meals, family history of pancreatic cancer, and age suggest chronic pancreatitis, a condition often associated with long-term inflammation of the pancreas.

HESI Exit Question 130: A 50-year-old male presents with recurrent abdominal pain, especially after consuming fatty meals. He also reports diarrhea and a history of alcohol abuse. What condition is most likely causing these symptoms?
A) Gastric ulcer
B) Pancreatitis
C) Gastroesophageal reflux disease (GERD)
D) Chronic pancreatitis

Rationale: The patient’s recurrent abdominal pain after consuming fatty meals, diarrhea, history of alcohol abuse, and signs of malabsorption are suggestive of chronic pancreatitis, a condition often associated with long-term inflammation of the pancreas.

HESI Exit Question 131: A 35-year-old female presents with upper abdominal pain, bloating, and diarrhea. She also reports a family history of cystic fibrosis. What condition should be suspected in this patient?

A) Gastric ulcer B) Pancreatitis C) Gastroesophageal reflux disease (GERD) D) Cystic fibrosis-related pancreatic insufficiency

Rationale: The patient’s upper abdominal pain, bloating, diarrhea, family history of cystic fibrosis, and age are suggestive of cystic fibrosis-related pancreatic insufficiency, which can lead to digestive symptoms.

HESI Exit Question 132: A 60-year-old male presents with upper abdominal pain, weight loss, and dark urine. He also reports a history of heavy alcohol consumption. What condition is most likely causing these symptoms?

A) Gastric ulcer B) Pancreatitis C) Gastroesophageal reflux disease (GERD) D) Alcoholic pancreatitis

Rationale: The patient’s upper abdominal pain, weight loss, dark urine, history of heavy alcohol consumption, and signs of pancreatitis suggest alcoholic pancreatitis, a specific form of pancreatitis related to alcohol abuse.

HESI Exit Question 133: A 45-year-old female presents with recurrent abdominal pain and diarrhea, especially after consuming fatty meals. She also reports a family history of pancreatic cancer. What condition should be suspected in this patient?

A) Gastric ulcer B) Pancreatitis C) Gastroesophageal reflux disease (GERD) D) Chronic pancreatitis

Rationale: The patient’s recurrent abdominal pain after consuming fatty meals, diarrhea, family history of pancreatic cancer, and age suggest chronic pancreatitis, a condition often associated with long-term inflammation of the pancreas.

HESI Exit Question 134: A 55-year-old male presents with upper abdominal pain, bloating, and diarrhea. He also reports a history of gallstones. What condition is most likely causing these symptoms?

A) Gastric ulcer B) Pancreatitis C) Gastroesophageal reflux disease (GERD) D) Chronic pancreatitis

Rationale: The patient’s upper abdominal pain, bloating, diarrhea, history of gallstones, and signs of malabsorption suggest chronic pancreatitis, which can be related to gallstone-induced pancreatitis.

HESI Exit Question 135: A 40-year-old male presents with recurrent upper abdominal pain, especially after consuming fatty meals. He also reports flatulence and diarrhea. What condition should be suspected in this patient?

A) Gastric ulcer B) Pancreatitis C) Gastroesophageal reflux disease (GERD) D) Chronic pancreatitis

Rationale: The patient’s recurrent upper abdominal pain after consuming fatty meals, flatulence, diarrhea, and signs of malabsorption suggest chronic pancreatitis, a condition often associated with long-term inflammation of the pancreas.

HESI Exit Question 136: A 30-year-old female presents with chronic abdominal pain, diarrhea, and weight loss. She also reports blood in her stool and fatigue. Laboratory tests reveal elevated inflammatory markers. What condition should be suspected in this patient?

A) Irritable bowel syndrome (IBS) B) Crohn’s disease C) Celiac disease D) Diverticulitis

Rationale: The patient’s chronic abdominal pain, diarrhea, weight loss, blood in stool, elevated inflammatory markers, and age are suggestive of Crohn’s disease, a type of inflammatory bowel disease (IBD).

HESI Exit Question 137: A 25-year-old male presents with recurrent abdominal cramps, bloating, and diarrhea, particularly during periods of stress. He also reports relief of symptoms after bowel movements. What condition is most likely causing these symptoms?

A) Irritable bowel syndrome (IBS) B) Crohn’s disease C) Celiac disease D) Diverticulitis

Rationale: The patient’s recurrent abdominal cramps, bloating, diarrhea related to stress, and relief of symptoms after bowel movements are suggestive of irritable bowel syndrome (IBS), a functional gastrointestinal disorder.

HESI Exit Question 138: A 35-year-old female presents with chronic diarrhea, abdominal pain, and weight loss. She also reports joint pain and skin rashes. Laboratory tests reveal elevated tissue transglutaminase antibodies. What condition should be suspected in this patient?

A) Irritable bowel syndrome (IBS) B) Crohn’s disease C) Celiac disease D) Diverticulitis

Rationale: The patient’s chronic diarrhea, abdominal pain, weight loss, joint pain, skin rashes, and elevated tissue transglutaminase antibodies are indicative of celiac disease, an autoimmune disorder triggered by gluten consumption.

HESI Exit Question 139: A 40-year-old male presents with abdominal pain, fever, and left lower quadrant tenderness. He also reports a history of constipation. What condition is most likely causing these symptoms?

A) Irritable bowel syndrome (IBS) B) Crohn’s disease C) Celiac disease D) Diverticulitis

Rationale: The patient’s abdominal pain, fever, left lower quadrant tenderness, and history of constipation are suggestive of diverticulitis, inflammation of the diverticula in the colon.

HESI Exit Question 140: A 45-year-old female presents with chronic diarrhea, abdominal pain, and weight loss. She also reports perianal fistulas and strictures. What condition should be suspected in this patient?

A) Irritable bowel syndrome (IBS) B) Crohn’s disease C) Celiac disease D) Diverticulitis

Correct Answer: B) Crohn’s disease

Rationale: The patient’s chronic diarrhea, abdominal pain, weight loss, perianal fistulas, and strictures are indicative of Crohn’s disease, a type of inflammatory bowel disease (IBD).

HESI Exit Question 141: A 30-year-old male presents with recurrent abdominal pain, bloating, and diarrhea, particularly after consuming dairy products. He also reports weight loss. What condition is most likely causing these symptoms?

A) Irritable bowel syndrome (IBS) B) Crohn’s disease C) Lactose intolerance D) Diverticulitis

Rationale: The patient’s recurrent abdominal pain, bloating, diarrhea after consuming dairy products, and weight loss are suggestive of lactose intolerance, which involves the inability to digest lactose.

HESI Exit Question 142: A 35-year-old female presents with chronic diarrhea, abdominal pain, and weight loss. She also reports a family history of Crohn’s disease. What condition should be suspected in this patient?

A) Irritable bowel syndrome (IBS) B) Crohn’s disease C) Celiac disease D) Diverticulitis

Rationale: The patient’s chronic diarrhea, abdominal pain, weight loss, family history of Crohn’s disease, and age are suggestive of Crohn’s disease, a type of inflammatory bowel disease (IBD).

HESI Exit Question 143: A 45-year-old male presents with recurrent abdominal cramps, bloating, and alternating diarrhea and constipation. He also reports relief of symptoms after bowel movements. What condition is most likely causing these symptoms?

A) Irritable bowel syndrome (IBS) B) Crohn’s disease C) Celiac disease D) Diverticulitis

Rationale: The patient’s recurrent abdominal cramps, bloating, alternating diarrhea and constipation, and relief of symptoms after bowel movements are suggestive of irritable bowel syndrome (IBS), a functional gastrointestinal disorder.

HESI Exit Question 144: A 40-year-old female presents with abdominal pain, diarrhea, and weight loss. She also reports mouth ulcers and erythema nodosum on her legs. What condition should be suspected in this patient?

A) Irritable bowel syndrome (IBS) B) Crohn’s disease C) Celiac disease D) Behçet’s syndrome

Rationale: The patient’s abdominal pain, diarrhea, weight loss, mouth ulcers, and erythema nodosum are indicative of Crohn’s disease, a type of inflammatory bowel disease (IBD).

HESI Exit Question 145: A 55-year-old male presents with recurrent abdominal cramps, bloating, and diarrhea, especially after consuming certain foods. He also reports fatigue and joint pain. What condition is most likely causing these symptoms?

A) Irritable bowel syndrome (IBS) B) Crohn’s disease C) Food allergies D) Diverticulitis

Rationale: The patient’s recurrent abdominal cramps, bloating, diarrhea after consuming certain foods, fatigue, and joint pain suggest food allergies, which can lead to gastrointestinal and systemic symptoms.

HESI Exit Question 146: A 50-year-old female presents with chronic diarrhea, abdominal pain, and weight loss. She also reports a history of heavy alcohol consumption. What condition should be suspected in this patient?

A) Irritable bowel syndrome (IBS) B) Crohn’s disease C) Alcohol-related gastrointestinal disorder D) Diverticulitis

Rationale: The patient’s chronic diarrhea, abdominal pain, weight loss, history of heavy alcohol consumption, and absence of specific findings for other conditions suggest alcohol-related gastrointestinal disorder.

HESI Exit Question 147: A 35-year-old male presents with recurrent abdominal pain, bloating, and alternating diarrhea and constipation. He also reports weight loss and a family history of colon cancer. What condition is most likely causing these symptoms?

A) Irritable bowel syndrome (IBS) B) Crohn’s disease C) Colon cancer D) Diverticulitis

Rationale: The patient’s recurrent abdominal pain, bloating, alternating diarrhea and constipation, weight loss, and family history of colon cancer raise concern for colon cancer, which can present with these symptoms.

HESI Exit Question 148: A 40-year-old female presents with chronic diarrhea, abdominal pain, and weight loss. She also reports mucous and blood in her stool. What condition should be suspected in this patient?

A) Irritable bowel syndrome (IBS) B) Crohn’s disease C) Ulcerative colitis D) Diverticulitis

Rationale: The patient’s chronic diarrhea, abdominal pain, weight loss, mucous and blood in stool are suggestive of ulcerative colitis, a type of inflammatory bowel disease (IBD).

HESI Exit Question 149: A 45-year-old male presents with recurrent abdominal cramps, bloating, and alternating diarrhea and constipation. He also reports relief of symptoms after bowel movements. What condition is most likely causing these symptoms?

A) Irritable bowel syndrome (IBS) B) Crohn’s disease C) Celiac disease D) Diverticulitis

Correct Answer: A) Irritable bowel syndrome (IBS)

Rationale: The patient’s recurrent abdominal cramps, bloating, alternating diarrhea and constipation, and relief of symptoms after bowel movements are suggestive of irritable bowel syndrome (IBS), a functional gastrointestinal disorder.

HESI Exit Question 150: A 50-year-old female presents with chronic diarrhea, abdominal pain, and weight loss. She also reports a history of heavy alcohol consumption. What condition should be suspected in this patient?

A) Irritable bowel syndrome (IBS) B) Crohn’s disease C) Alcohol-related gastrointestinal disorder D) Diverticulitis

Rationale: The patient’s chronic diarrhea, abdominal pain, weight loss, history of heavy alcohol consumption, and absence of specific findings for other conditions suggest alcohol-related gastrointestinal disorder.

HESI Exit Question 151: A 65-year-old male presents with infrequent, hard stools and a sensation of incomplete evacuation. He also reports abdominal discomfort and a history of chronic use of pain medication. What condition is most likely causing these symptoms?

A) Diarrhea-predominant irritable bowel syndrome (IBS) B) Constipation C) Diverticulitis D) Gastroenteritis

Rationale: The patient’s infrequent, hard stools, sensation of incomplete evacuation, abdominal discomfort, and history of chronic use of pain medication use are suggestive of constipation.

HESI Exit Question 152: A 30-year-old female presents with frequent, watery stools and abdominal cramps. She also reports fever and nausea. Laboratory tests reveal leukocytosis. What condition should be suspected in this patient?

A) Diarrhea-predominant irritable bowel syndrome (IBS) B) Constipation C) Diverticulitis D) Gastroenteritis

Rationale: The patient’s frequent, watery stools, abdominal cramps, fever, nausea, and leukocytosis are indicative of gastroenteritis, an infectious cause of diarrhea.

HESI Exit Question 153: A 40-year-old male presents with recurrent, crampy abdominal pain, alternating between diarrhea and constipation. He also reports relief of symptoms after bowel movements. What condition is most likely causing these symptoms?

A) Diarrhea-predominant irritable bowel syndrome (IBS) B) Constipation C) Diverticulitis D) Gastroenteritis

Rationale: The patient’s recurrent, crampy abdominal pain, alternating between diarrhea and constipation, and relief of symptoms after bowel movements are suggestive of diarrhea-predominant irritable bowel syndrome (IBS).

HESI Exit Question 154: A 50-year-old female presents with infrequent, hard stools and a sensation of incomplete evacuation. She also reports abdominal discomfort and a family history of colon cancer. What condition should be suspected in this patient?

A) Diarrhea-predominant irritable bowel syndrome (IBS) B) Constipation C) Diverticulitis D) Colon cancer

Rationale: The patient’s infrequent, hard stools, sensation of incomplete evacuation, abdominal discomfort, and family history of colon cancer raise concern for constipation.

HESI Exit Question 155: A 35-year-old male presents with frequent, watery stools and abdominal cramps. He also reports a history of lactose intolerance. What condition is most likely causing these symptoms?

A) Diarrhea-predominant irritable bowel syndrome (IBS) B) Constipation C) Lactose intolerance D) Gastroenteritis

Rationale: The patient’s frequent, watery stools, abdominal cramps, history of lactose intolerance, and absence of fever or leukocytosis suggest lactose intolerance as the likely cause.

HESI Exit Question 156: A 45-year-old female presents with recurrent, crampy abdominal pain, alternating between diarrhea and constipation. She also reports relief of symptoms after bowel movements. What condition should be suspected in this patient?

A) Diarrhea-predominant irritable bowel syndrome (IBS) B) Constipation C) Diverticulitis D) Gastroenteritis

Rationale: The patient’s recurrent, crampy abdominal pain, alternating between diarrhea and constipation, and relief of symptoms after bowel movements are suggestive of diarrhea-predominant irritable bowel syndrome (IBS).

HESI Exit Question 157: A 60-year-old male presents with infrequent, hard stools and a sensation of incomplete evacuation. He also reports blood in his stool and unexplained weight loss. What condition is most likely causing these symptoms?

A) Diarrhea-predominant irritable bowel syndrome (IBS) B) Constipation C) Diverticulitis D) Colorectal cancer

Rationale: The patient’s infrequent, hard stools, sensation of incomplete evacuation, blood in stool, unexplained weight loss, and age raise concern for colorectal cancer.

HESI Exit Question 158: A 55-year-old female presents with frequent, watery stools and abdominal cramps. She also reports a history of inflammatory bowel disease (IBD). What condition should be suspected in this patient?

A) Diarrhea-predominant irritable bowel syndrome (IBS) B) Constipation C) IBD flare D) Gastroenteritis

Rationale: The patient’s frequent, watery stools, abdominal cramps, history of inflammatory bowel disease (IBD), and absence of infectious symptoms suggest an IBD flare.

HESI Exit Question 160: A 65-year-old male presents with fatigue, shortness of breath, and peripheral edema. Laboratory tests reveal elevated serum creatinine and blood urea nitrogen (BUN) levels. His urine output has decreased significantly. What condition should be suspected in this patient?

A) Urinary tract infection (UTI) B) Acute kidney injury (AKI) C) Chronic kidney disease (CKD) D) Nephrotic syndrome

Rationale: The patient’s fatigue, shortness of breath, peripheral edema, elevated serum creatinine, elevated BUN, and decreased urine output are indicative of chronic kidney disease (CKD).

HESI Exit Question 161: A 45-year-old female presents with hypertension, proteinuria, and swelling in her ankles and face. Laboratory tests reveal hypoalbuminemia and elevated cholesterol levels. What condition should be suspected in this patient?

A) Urinary tract infection (UTI) B) Acute kidney injury (AKI) C) Chronic kidney disease (CKD) D) Nephrotic syndrome

Correct Answer: D) Nephrotic syndrome

Rationale: The patient’s hypertension, proteinuria, swelling, hypoalbuminemia, and elevated cholesterol levels are suggestive of nephrotic syndrome, a kidney disorder characterized by glomerular damage.

HESI Exit Question 162: A 35-year-old male presents with flank pain, hematuria, and a history of kidney stones. Imaging reveals a 6mm stone in the right renal pelvis. What is the most likely diagnosis in this patient?

A) Urinary tract infection (UTI) B) Acute kidney injury (AKI) C) Chronic kidney disease (CKD) D) Renal calculi (kidney stones)

Correct Answer: D) Renal calculi (kidney stones)

Rationale: The patient’s flank pain, hematuria, and imaging findings of a kidney stone in the renal pelvis point to a diagnosis of renal calculi or kidney stones.

HESI Exit Question 163: A 55-year-old female presents with nocturia, polyuria, and excessive thirst. Laboratory tests reveal elevated serum glucose levels and glycosuria. What condition should be suspected in this patient?

A) Urinary tract infection (UTI) B) Acute kidney injury (AKI) C) Chronic kidney disease (CKD) D) Diabetes mellitus (DM)

Correct Answer: D) Diabetes mellitus (DM)

Rationale: The patient’s nocturia, polyuria, excessive thirst, elevated serum glucose levels, and glycosuria are indicative of diabetes mellitus (DM), a metabolic disorder that can lead to kidney complications.

HESI Exit Question 164: A 40-year-old male presents with urgency, frequency, and dysuria. He also reports fever and flank pain. Laboratory tests reveal pyuria and bacteriuria. What condition should be suspected in this patient?

A) Urinary tract infection (UTI) B) Acute kidney injury (AKI) C) Chronic kidney disease (CKD) D) Glomerulonephritis

Correct Answer: A) Urinary tract infection (UTI)

Rationale: The patient’s urgency, frequency, dysuria, fever, flank pain, pyuria, and bacteriuria are indicative of a urinary tract infection (UTI).

HESI Exit Question 165: A 50-year-old female presents with fatigue, confusion, and muscle weakness. Laboratory tests reveal elevated serum phosphate and low serum calcium levels. What condition should be suspected in this patient?

A) Urinary tract infection (UTI) B) Acute kidney injury (AKI) C) Hyperparathyroidism D) Hypoparathyroidism

Correct Answer: C) Hyperparathyroidism

Rationale: The patient’s fatigue, confusion, muscle weakness, elevated serum phosphate, and low serum calcium levels suggest hyperparathyroidism, often associated with kidney dysfunction.

HESI Exit Question 166: A 45-year-old male presents with oliguria, dark urine, and jaundice. Laboratory tests reveal elevated serum bilirubin and elevated creatinine. What condition should be suspected in this patient?

A) Urinary tract infection (UTI) B) Acute kidney injury (AKI) C) Hepatorenal syndrome D) Hemolytic-uremic syndrome (HUS)

Correct Answer: C) Hepatorenal syndrome

Rationale: The patient’s oliguria, dark urine, jaundice, elevated serum bilirubin, and elevated creatinine are suggestive of hepatorenal syndrome, a complication of severe liver disease.

HESI Exit Question 167: A 35-year-old female presents with hypertension, fatigue, and lower extremity edema. Laboratory tests reveal hematuria and proteinuria. What condition should be suspected in this patient?

A) Urinary tract infection (UTI) B) Acute kidney injury (AKI) C) Chronic kidney disease (CKD) D) Glomerulonephritis

Correct Answer: D) Glomerulonephritis

Rationale: The patient’s hypertension, fatigue, lower extremity edema, hematuria, and proteinuria are indicative of glomerulonephritis, an inflammation of the glomeruli in the kidneys.

HESI Exit Question 168: A 60-year-old male presents with confusion, muscle twitches, and tingling sensations. Laboratory tests reveal elevated serum phosphate and low serum calcium levels. What condition should be suspected in this patient?

A) Urinary tract infection (UTI) B) Acute kidney injury (AKI) C) Hyperparathyroidism D) Hypoparathyroidism

Correct Answer: D) Hypoparathyroidism

Rationale: The patient’s confusion, muscle twitches, tingling sensations, elevated serum phosphate, and low serum calcium levels are suggestive of hypoparathyroidism, often associated with kidney dysfunction.

HESI Exit Question 169: A 55-year-old female presents with polyuria, polydipsia, and hypotension. Laboratory tests reveal hyponatremia and hyperkalemia. What condition should be suspected in this patient?

A) Urinary tract infection (UTI) B) Acute kidney injury (AKI) C) Diabetes insipidus (DI) D) Addison’s disease

Correct Answer: D) Addison’s disease

Rationale: The patient’s polyuria, polydipsia, hypotension, hyponatremia, and hyperkalemia are indicative of Addison’s disease, an endocrine disorder affecting the adrenal glands, which can be associated with kidney dysfunction.

HESI Exit Question 170: A 65-year-old male presents with recurrent urinary tract infections, dribbling, and difficulty starting and stopping urination. Physical examination reveals an enlarged prostate. What condition should be suspected in this patient?

A) Urinary tract infection (UTI) B) Acute kidney injury (AKI) C) Benign prostatic hyperplasia (BPH) D) Prostate cancer

Correct Answer: C) Benign prostatic hyperplasia (BPH)

Rationale: The patient’s recurrent urinary tract infections, dribbling, difficulty starting and stopping urination, and an enlarged prostate are suggestive of benign prostatic hyperplasia (BPH).

HESI Exit Question 171: A 70-year-old male presents with confusion, nausea, and vomiting. Laboratory tests reveal elevated serum creatinine and blood urea nitrogen (BUN) levels. His urine output has significantly decreased. What condition should be suspected in this patient?

A) Urinary tract infection (UTI) B) Acute kidney injury (AKI) C) Chronic kidney disease (CKD) D) Dehydration

Correct Answer: B) Acute kidney injury (AKI)

Rationale: The patient’s confusion, nausea, vomiting, elevated serum creatinine, elevated BUN, and significantly decreased urine output are indicative of acute kidney injury (AKI), often associated with dehydration.

HESI Exit Question 172: A 50-year-old female presents with hematuria, flank pain, and a palpable mass in the abdomen. Imaging reveals a solid mass in the right kidney. What is the most likely diagnosis in this patient?

A) Urinary tract infection (UTI) B) Acute kidney injury (AKI) C) Renal cell carcinoma D) Pyelonephritis

Correct Answer: C) Renal cell carcinoma

Rationale: The patient’s hematuria, flank pain, palpable abdominal mass, and imaging findings of a solid mass in the right kidney are indicative of renal cell carcinoma, a type of kidney cancer.

HESI Exit Question 173: A 45-year-old male presents with fatigue, anorexia, and pruritus. Laboratory tests reveal elevated serum bilirubin and elevated creatinine. What condition should be suspected in this patient?

A) Urinary tract infection (UTI) B) Acute kidney injury (AKI) C) Hepatorenal syndrome D) Hemolytic-uremic syndrome (HUS)

Correct Answer: C) Hepatorenal syndrome

Rationale: The patient’s fatigue, anorexia, pruritus, elevated serum bilirubin, and elevated creatinine are suggestive of hepatorenal syndrome, a complication of severe liver disease.

HESI Exit Question 174: A 35-year-old female presents with hypertension, headache, and generalized edema. Laboratory tests reveal hypoalbuminemia and hyperlipidemia. What condition should be suspected in this patient?

A) Urinary tract infection (UTI) B) Acute kidney injury (AKI) C) Chronic kidney disease (CKD) D) Nephrotic syndrome

Correct Answer: D) Nephrotic syndrome

Rationale: The patient’s hypertension, headache, generalized edema, hypoalbuminemia, and hyperlipidemia are indicative of nephrotic syndrome, a kidney disorder characterized by glomerular damage.

HESI Exit Question 175: A 60-year-old male presents with oliguria, dark urine, and jaundice. Laboratory tests reveal elevated serum bilirubin and elevated creatinine. What condition should be suspected in this patient?

A) Urinary tract infection (UTI) B) Acute kidney injury (AKI) C) Hepatorenal syndrome D) Hemolytic-uremic syndrome (HUS)

Correct Answer: C) Hepatorenal syndrome

Rationale: The patient’s oliguria, dark urine, jaundice, elevated serum bilirubin, and elevated creatinine are suggestive of hepatorenal syndrome, a complication of severe liver disease.

HESI Exit Question 176: A 40-year-old female presents with polyuria, polydipsia, and hypertension. Laboratory tests reveal proteinuria and hematuria. What condition should be suspected in this patient?

A) Urinary tract infection (UTI) B) Acute kidney injury (AKI) C) Diabetic nephropathy D) Glomerulonephritis

Correct Answer: C) Diabetic nephropathy

Rationale: The patient’s polyuria, polydipsia, hypertension, proteinuria, hematuria, and history of diabetes mellitus are indicative of diabetic nephropathy, a kidney complication of diabetes.

HESI Exit Question 177: A 50-year-old male presents with recurrent urinary tract infections, dribbling, and difficulty starting and stopping urination. Physical examination reveals an enlarged prostate. What condition should be suspected in this patient?

A) Urinary tract infection (UTI) B) Acute kidney injury (AKI) C) Benign prostatic hyperplasia (BPH) D) Prostate cancer

Correct Answer: C) Benign prostatic hyperplasia (BPH)

Rationale: The patient’s recurrent urinary tract infections, dribbling, difficulty starting and stopping urination, and an enlarged prostate are suggestive of benign prostatic hyperplasia (BPH).

HESI Exit Question 178: A 70-year-old male presents with confusion, nausea, and vomiting. Laboratory tests reveal elevated serum creatinine and blood urea nitrogen (BUN) levels. His urine output has significantly decreased. What condition should be suspected in this patient?

A) Urinary tract infection (UTI) B) Acute kidney injury (AKI) C) Chronic kidney disease (CKD) D) Dehydration

Correct Answer: B) Acute kidney injury (AKI)

Rationale: The patient’s confusion, nausea, vomiting, elevated serum creatinine, elevated BUN, and significantly decreased urine output are indicative of acute kidney injury (AKI), often associated with dehydration.

HESI Exit Question 179: A 55-year-old female presents with hematuria, flank pain, and a palpable mass in the abdomen. Imaging reveals a solid mass in the right kidney. What is the most likely diagnosis in this patient?

A) Urinary tract infection (UTI) B) Acute kidney injury (AKI) C) Renal cell carcinoma D) Pyelonephritis

Correct Answer: C) Renal cell carcinoma

Rationale: The patient’s hematuria, flank pain, palpable abdominal mass, and imaging findings of a solid mass in the right kidney are indicative of renal cell carcinoma, a type of kidney cancer.

HESI Exit Question 180: A 25-year-old female presents with a pruritic, erythematous rash on her arms and legs. She reports recent exposure to poison ivy while camping. What is the most likely diagnosis?

A) Atopic dermatitis B) Contact dermatitis C) Psoriasis D) Scabies

Correct Answer: B) Contact dermatitis

Rationale: The patient’s pruritic, erythematous rash following exposure to poison ivy is characteristic of contact dermatitis, an inflammatory skin reaction to an irritant.

HESI Exit Question 181: A 30-year-old male presents with flaky, silvery-white plaques on his elbows and knees. He reports a family history of the same condition. What condition should be suspected in this patient?

A) Atopic dermatitis B) Contact dermatitis C) Psoriasis D) Scabies

Correct Answer: C) Psoriasis

Rationale: The patient’s flaky, silvery-white plaques on elbows and knees, along with a family history, are indicative of psoriasis, a chronic autoimmune skin condition.

HESI Exit Question 182: A 40-year-old female presents with small, fluid-filled vesicles on her hands and fingers. She reports intense itching and a history of similar episodes. What condition should be suspected in this patient?

A) Atopic dermatitis B) Contact dermatitis C) Psoriasis D) Dyshidrotic eczema

Correct Answer: D) Dyshidrotic eczema

Rationale: The patient’s small, fluid-filled vesicles on hands and fingers, intense itching, and recurrent nature are suggestive of dyshidrotic eczema, a type of eczema affecting the hands and feet.

HESI Exit Question 183: A 35-year-old male presents with erythematous, scaly patches on his face and scalp. He reports a history of similar episodes and mentions that his father had the same condition. What condition should be suspected in this patient?

A) Atopic dermatitis B) Contact dermatitis C) Psoriasis D) Rosacea

Correct Answer: C) Psoriasis

Rationale: The patient’s erythematous, scaly patches on the face and scalp, recurrent episodes, and family history are indicative of psoriasis, a chronic autoimmune skin condition.

HESI Exit Question 184: A 45-year-old male presents with intensely pruritic, linear burrows on his hands and wrists. He reports recent camping activities and sleeping in a communal cabin. What condition should be suspected in this patient?

A) Atopic dermatitis B) Contact dermatitis C) Psoriasis D) Scabies

Correct Answer: D) Scabies

Rationale: The patient’s intensely pruritic, linear burrows following camping activities and communal sleeping are suggestive of scabies, a parasitic skin infestation.

HESI Exit Question 185: A 50-year-old female presents with persistent redness, flushing, and visible blood vessels on her face, particularly on the cheeks and nose. What condition should be suspected in this patient?

A) Atopic dermatitis B) Contact dermatitis C) Psoriasis D) Rosacea

Correct Answer: D) Rosacea

Rationale: The patient’s persistent redness, flushing, and visible blood vessels on the face are characteristic of rosacea, a chronic skin condition.

HESI Exit Question 186: A 55-year-old male presents with painful, fluid-filled vesicles on his chest and back, following a recent episode of shingles. What is the most likely diagnosis?

A) Herpes simplex virus (HSV) infection B) Tinea corporis (ringworm) C) Herpes zoster (shingles) D) Scabies

Correct Answer: C) Herpes zoster (shingles)

Rationale: The patient’s painful, fluid-filled vesicles on the chest and back following a recent episode of shingles are indicative of herpes zoster, caused by the varicella-zoster virus.

HESI Exit Question 187: A 60-year-old male presents with a dark, irregularly shaped mole on his back that has increased in size and changed in color over the past year. What condition should be suspected in this patient?

A) Atopic dermatitis B) Contact dermatitis C) Psoriasis D) Melanoma

Correct Answer: D) Melanoma

Rationale: The patient’s description of a dark, irregularly shaped mole that has increased in size and changed in color is concerning for melanoma, a potentially aggressive form of skin cancer.

HESI Exit Question 188: A 65-year-old female presents with thick, scaly, and erythematous patches on her lower legs. She reports itching and a history of similar episodes during the winter months. What condition should be suspected in this patient?

A) Atopic dermatitis B) Contact dermatitis C) Psoriasis D) Xerosis

Correct Answer: D) Xerosis

Rationale: The patient’s thick, scaly, erythematous patches on the lower legs during the winter months are characteristic of xerosis, or dry skin.

HESI Exit Question 189: A 70-year-old male presents with a painless, waxy nodule on his face. He reports it has been slowly growing over the past few years. What condition should be suspected in this patient?

A) Atopic dermatitis B) Contact dermatitis C) Psoriasis D) Basal cell carcinoma

Correct Answer: D) Basal cell carcinoma

Rationale: The patient’s painless, waxy nodule that has been slowly growing is indicative of basal cell carcinoma, a common type of skin cancer.

HESI Exit Question 190: A 75-year-old female presents with a rough, scaly, and erythematous lesion on her lower lip. She reports chronic sun exposure and a history of tobacco use. What condition should be suspected in this patient?

A) Atopic dermatitis B) Contact dermatitis C) Psoriasis D) Squamous cell carcinoma

Correct Answer: D) Squamous cell carcinoma

Rationale: The patient’s rough, scaly, erythematous lesion on the lower lip, chronic sun exposure, and history of tobacco use are indicative of squamous cell carcinoma, a type of skin cancer.

HESI Exit Question 191: An 80-year-old male presents with pain, redness, and swelling of his left great toe. He reports a recent seafood feast and a history of gout. What condition should be suspected in this patient?

A) Atopic dermatitis B) Contact dermatitis C) Psoriasis D) Gouty arthritis

Correct Answer: D) Gouty arthritis

Rationale: The patient’s pain, redness, swelling of the left great toe, recent seafood feast, and history of gout are suggestive of gouty arthritis, a type of inflammatory arthritis.

HESI Exit Question 231: A full-term newborn presents with cyanosis, poor feeding, and respiratory distress shortly after birth. On examination, there is a continuous machine-like murmur heard over the left upper sternal border. What congenital heart defect should be suspected in this neonate?

A) Ventricular septal defect (VSD) B) Atrial septal defect (ASD) C) Tetralogy of Fallot (TOF) D) Patent ductus arteriosus (PDA)

Correct Answer: D) Patent ductus arteriosus (PDA)

Rationale: The continuous machine-like murmur over the left upper sternal border, along with cyanosis and respiratory distress, is indicative of a patent ductus arteriosus (PDA), a congenital heart defect.

HESI Exit Question 232: A term newborn exhibits jitteriness, poor feeding, and a high-pitched cry. The infant’s mother reports a history of alcohol abuse during pregnancy. What condition should be suspected in this neonate?

A) Hypoglycemia B) Hypocalcemia C) Neonatal abstinence syndrome (NAS) D) Hyperbilirubinemia

Correct Answer: C) Neonatal abstinence syndrome (NAS)

Rationale: The newborn’s jitteriness, poor feeding, and high-pitched cry, along with the maternal history of alcohol abuse, are suggestive of neonatal abstinence syndrome (NAS) due to prenatal drug exposure.

HESI Exit Question 233: A newborn presents with bilious vomiting, abdominal distention, and failure to pass meconium. What congenital gastrointestinal disorder should be suspected in this neonate?

A) Intussusception B) Hirschsprung’s disease C) Gastroesophageal reflux disease (GERD) D) Necrotizing enterocolitis (NEC)

Correct Answer: B) Hirschsprung’s disease

Rationale: The newborn’s symptoms of bilious vomiting, abdominal distention, and failure to pass meconium are indicative of Hirschsprung’s disease, a congenital disorder characterized by a lack of ganglion cells in the rectum and lower colon.

HESI Exit Question 234: A preterm neonate born at 28 weeks gestation presents with respiratory distress, grunting, nasal flaring, and retractions. What condition should be suspected in this neonate?

A) Respiratory distress syndrome (RDS) B) Bronchopulmonary dysplasia (BPD) C) Meconium aspiration syndrome (MAS) D) Transient tachypnea of the newborn (TTN)

Correct Answer: A) Respiratory distress syndrome (RDS)

Rationale: The preterm neonate’s symptoms of respiratory distress, grunting, nasal flaring, and retractions are indicative of respiratory distress syndrome (RDS), a common complication in preterm infants due to immature lung development.

HESI Exit Question 235: A term neonate presents with jaundice within the first 24 hours of life. Physical examination reveals hepatomegaly and an enlarged tongue. What condition should be suspected in this neonate?

A) Physiological jaundice B) Hemolytic disease of the newborn (HDN) C) Hypothyroidism D) Galactosemia

Correct Answer: D) Galactosemia

Rationale: The neonate’s early onset jaundice, hepatomegaly, and enlarged tongue are indicative of galactosemia, a metabolic disorder characterized by an inability to metabolize galactose.

HESI Exit Question 236: A full-term neonate presents with fever, irritability, poor feeding, and a bulging fontanelle. What condition should be suspected in this neonate?

A) Meningitis B) Hypoglycemia C) Neonatal sepsis D) Hypocalcemia

Correct Answer: A) Meningitis

Rationale: The neonate’s symptoms of fever, irritability, poor feeding, and a bulging fontanelle raise concern for meningitis, a serious infection of the central nervous system.

HESI Exit Question 237: A newborn presents with cyanosis, tachypnea, and grunting shortly after birth. On examination, there is decreased breath sounds on one side of the chest. What condition should be suspected in this neonate?

A) Pneumothorax B) Meconium aspiration syndrome (MAS) C) Respiratory distress syndrome (RDS) D) Transient tachypnea of the newborn (TTN)

Correct Answer: A) Pneumothorax

Rationale: The newborn’s symptoms of cyanosis, tachypnea, grunting, decreased breath sounds on one side of the chest, and a risk factor for mechanical ventilation suggest pneumothorax.

HESI Exit Question 238: A term neonate presents with poor feeding, vomiting, and irritability. Physical examination reveals an olive-shaped mass in the right upper quadrant of the abdomen. What condition should be suspected in this neonate?

A) Hypertrophic pyloric stenosis (HPS) B) Gastroesophageal reflux disease (GERD) C) Necrotizing enterocolitis (NEC) D) Meconium ileus

Correct Answer: A) Hypertrophic pyloric stenosis (HPS)

Rationale: The neonate’s symptoms of poor feeding, vomiting, irritability, and an olive-shaped mass in the right upper quadrant of the abdomen are indicative of hypertrophic pyloric stenosis (HPS).

HESI Exit Question 239: A newborn presents with cyanosis, apnea, and poor muscle tone shortly after birth. What condition should be suspected in this neonate?

A) Hypoglycemia B) Hypocalcemia C) Transient tachypnea of the newborn (TTN) D) Neonatal respiratory distress syndrome (NRDS)

Correct Answer: D) Neonatal respiratory distress syndrome (NRDS)

Rationale: The newborn’s symptoms of cyanosis, apnea, and poor muscle tone shortly after birth are indicative of neonatal respiratory distress syndrome (NRDS), also known as hyaline membrane disease.

HESI Exit Question 240: A term newborn presents with a “strawberry” birthmark on the face. What is the term for this type of birthmark?

A) Nevus sebaceous B) Hemangioma C) Café-au-lait spot D) Mongolian spot

Correct Answer: B) Hemangioma

Rationale: The “strawberry” birthmark on the face is a type of hemangioma, a common benign vascular birthmark in newborns.

HESI Exit Question 241: A preterm neonate presents with abdominal distention, bloody stools, and signs of shock. What condition should be suspected in this neonate?

A) Meconium aspiration syndrome (MAS) B) Necrotizing enterocolitis (NEC) C) Gastroesophageal reflux disease (GERD) D) Transient tachypnea of the newborn (TTN)

Correct Answer: B) Necrotizing enterocolitis (NEC)

Rationale: The preterm neonate’s symptoms of abdominal distention, bloody stools, and signs of shock are indicative of necrotizing enterocolitis (NEC), a serious gastrointestinal condition.

HESI Exit Question 242: A newborn presents with a yellowish discoloration of the skin and sclera within the first 24 hours of life. What is the term for this type of jaundice?

A) Physiological jaundice B) Hemolytic jaundice C) Breast milk jaundice D) Pathological jaundice

Correct Answer: D) Pathological jaundice

Rationale: Jaundice occurring within the first 24 hours of life is termed pathological jaundice and requires prompt evaluation for underlying causes.

HESI Exit Question 243: A term newborn presents with respiratory distress, grunting, and nasal flaring. The infant’s mother has a history of gestational diabetes. What condition should be suspected in this neonate?

A) Respiratory distress syndrome (RDS) B) Bronchopulmonary dysplasia (BPD) C) Meconium aspiration syndrome (MAS) D) Transient tachypnea of the newborn (TTN)

Correct Answer: B) Bronchopulmonary dysplasia (BPD)

Rationale: The newborn’s respiratory distress, maternal history of gestational diabetes, and risk factors suggest bronchopulmonary dysplasia (BPD), a chronic lung disease of prematurity.

HESI Exit Question 244: A term neonate presents with poor muscle tone, hypotonia, and a weak cry. What metabolic disorder should be suspected in this neonate?

A) Phenylketonuria (PKU) B) Hypoglycemia C) Hypocalcemia D) Hypothyroidism

Correct Answer: D) Hypothyroidism

Rationale: The neonate’s symptoms of poor muscle tone, hypotonia, and a weak cry are suggestive of hypothyroidism, a metabolic disorder.

HESI Exit Question 245: A preterm neonate presents with apnea, bradycardia, and cyanosis. What condition should be suspected in this neonate?

A) Apnea of prematurity B) Transient tachypnea of the newborn (TTN) C) Hypoglycemia D) Neonatal sepsis

Correct Answer: A) Apnea of prematurity

Rationale: The preterm neonate’s symptoms of apnea, bradycardia, and cyanosis are characteristic of apnea of prematurity, which is common in preterm infants.

HESI Exit Question 246: A 2-month-old infant presents with poor head control, inability to roll over, and minimal social interactions. Which developmental milestone is the infant failing to achieve?

A) Crawling B) Sitting independently C) Grasping objects D) Lifting head while on the stomach

Correct Answer: D) Lifting head while on the stomach

Rationale: By 2 months of age, infants should be able to lift their heads while lying on their stomachs. This milestone is essential for building neck and upper body strength, which is a prerequisite for other developmental milestones.

HESI Exit Question 247: A 7-month-old infant presents with the inability to sit independently or bear weight on the legs. Which developmental milestone is the infant failing to achieve?

A) Crawling B) Rolling over C) Pulling to stand D) Babbling

Correct Answer: C) Pulling to stand

Rationale: At 7 months of age, infants should typically be able to pull themselves up to a standing position. The inability to bear weight on the legs and stand could indicate a delay in motor development.

HESI Exit Question 248: A 10-month-old infant presents with the inability to grasp small objects using the thumb and forefinger. Which developmental milestone is the infant failing to achieve?

A) Crawling B) Sitting independently C) Rolling over D) Pincer grasp

Correct Answer: D) Pincer grasp

Rationale: By 10 months of age, infants should develop the pincer grasp, which allows them to pick up small objects using the thumb and forefinger. Difficulty with this milestone can suggest a fine motor delay.

HESI Exit Question 249: A 3-year-old child presents with limited vocabulary and difficulty forming sentences. Which developmental milestone is the child failing to achieve?

A) Walking independently B) Recognizing colors C) Following simple instructions D) Expressive language development

Correct Answer: D) Expressive language development

Rationale: By the age of 3, children typically have a more extensive vocabulary and can form sentences. Delayed expressive language development may indicate a speech or language disorder.

HESI Exit Question 250: A 5-year-old child presents with difficulty recognizing letters and numbers. Which developmental milestone is the child failing to achieve?

A) Walking independently B) Reading readiness C) Counting to ten D) Basic addition and subtraction

Correct Answer: B) Reading readiness

Rationale: By the age of 5, children should exhibit reading readiness skills, which include recognizing letters and numbers. Difficulty in this area may indicate a potential learning disability.

HESI Exit Question 251: A 15-month-old toddler presents with difficulty walking and frequent falls. Which developmental milestone is the toddler failing to achieve?

A) Crawling B) Running C) Standing on one foot D) Walking independently

Correct Answer: D) Walking independently

Rationale: By 15 months of age, toddlers should be able to walk independently without frequent falls. Delayed walking can be a concern for motor development.

HESI Exit Question 252: A 4-year-old child presents with difficulty recognizing shapes and colors. Which developmental milestone is the child failing to achieve?

A) Drawing basic shapes B) Recognizing and naming shapes and colors C) Basic counting skills D) Reading readiness

Correct Answer: B) Recognizing and naming shapes and colors

Rationale: By the age of 4, children should be able to recognize and name basic shapes and colors. Difficulty in this area may indicate a developmental delay.

HESI Exit Question 253: A 6-year-old child presents with difficulty counting to ten and recognizing numbers. Which developmental milestone is the child failing to achieve?

A) Basic addition and subtraction B) Recognizing and naming numbers C) Reading readiness D) Understanding time concepts

Correct Answer: B) Recognizing and naming numbers

Rationale: By the age of 6, children should be able to recognize and name numbers. Difficulty in this area may suggest a delay in math-related developmental milestones.

HESI Exit Question 254: A 2-year-old toddler presents with a limited vocabulary and difficulty following simple instructions. Which developmental milestone is the toddler failing to achieve?

A) Crawling B) Running C) Expressive language development D) Pincer grasp

Correct Answer: C) Expressive language development

Rationale: By the age of 2, toddlers typically have a more extensive vocabulary and can follow simple instructions. Delayed expressive language development may indicate a speech or language disorder.

HESI Exit Question 255: A 8-year-old child presents with difficulty with basic addition and subtraction. Which developmental milestone is the child failing to achieve?

A) Basic multiplication and division B) Understanding time concepts C) Reading readiness D) Basic addition and subtraction

Correct Answer: D) Basic addition and subtraction

Rationale: By the age of 8, children should have a grasp of basic addition and subtraction. Difficulty in this area may suggest a developmental delay in math-related skills.

HESI Exit Question 256: A 1-year-old toddler presents with limited social interactions, poor eye contact, and a lack of response to their name. What developmental milestone should be assessed in this child?

A) Crawling B) Expressive language development C) Social-emotional development D) Pincer grasp

Correct Answer: C) Social-emotional development

Rationale: Limited social interactions, poor eye contact, and a lack of response to their name may raise concerns about social-emotional development and the possibility of autism spectrum disorder (ASD).

HESI Exit Question 257: A 7-year-old child presents with difficulty understanding time concepts, such as days of the week and months of the year. What developmental milestone is the child struggling with?

A) Basic addition and subtraction B) Reading readiness C) Recognizing and naming numbers D) Understanding time concepts

Correct Answer: D) Understanding time concepts

Rationale: By the age of 7, children should begin to understand time concepts, such as days of the week and months of the year. Difficulty in this area may indicate a delay in cognitive development.

HESI Exit Question 258: A 9-year-old child presents with difficulty reading and struggles with recognizing letters and words. What developmental milestone is the child failing to achieve?

A) Counting to ten B) Reading readiness C) Basic addition and subtraction D) Recognizing and naming shapes and colors

Correct Answer: B) Reading readiness

Rationale: By the age of 9, children should have developed reading readiness skills, including recognizing letters and words. Difficulty in this area may suggest a reading disorder.

HESI Exit Question 259: A 3-year-old child presents with frequent temper tantrums, difficulty sharing with peers, and aggression. What developmental milestone should be assessed in this child?

A) Social-emotional development B) Expressive language development C) Fine motor skills D) Basic counting skills

Correct Answer: A) Social-emotional development

Rationale: Frequent temper tantrums, difficulty sharing with peers, and aggression may indicate challenges in social-emotional development and the need for behavioral assessment.

HESI Exit Question 260: A 5-year-old child presents with difficulty with basic counting skills and struggles to count objects accurately. What developmental milestone is the child failing to achieve?

A) Reading readiness B) Basic addition and subtraction C) Recognizing and naming numbers D) Basic multiplication and division

Correct Answer: B) Basic addition and subtraction

Rationale: By the age of 5, children should have a basic understanding of counting objects accurately and performing basic addition and subtraction. Difficulty in this area may suggest a developmental delay in math-related skills.

HESI Exit Question 261: A 45-year-old patient presents with fatigue, pale skin, and a history of a vegetarian diet without supplementation. Laboratory tests reveal low hemoglobin and serum iron levels. What nutrient deficiency is most likely contributing to these symptoms?

A) Vitamin C deficiency B) Vitamin B12 deficiency C) Iron deficiency D) Calcium deficiency

Correct Answer: B) Vitamin B12 deficiency

Rationale: The patient’s fatigue, pale skin, and low hemoglobin and serum iron levels are indicative of anemia, often caused by vitamin B12 deficiency, which can be common in individuals following a strict vegetarian diet without supplementation.

HESI Exit Question 262: A 30-year-old pregnant woman is advised to increase her folate intake to prevent neural tube defects in her developing baby. Which of the following food sources is rich in folate?

A) Red meat B) Eggs C) Leafy greens D) Dairy products

Correct Answer: C) Leafy greens

Rationale: Leafy greens, such as spinach and kale, are rich sources of folate (also known as vitamin B9), which is essential for preventing neural tube defects during pregnancy.

HESI Exit Question 263: A 60-year-old patient with a history of osteoporosis is advised to increase dietary calcium intake. Which of the following food sources is an excellent source of calcium?

A) Chicken breast B) Whole grains C) Broccoli D) Milk

Correct Answer: D) Milk

Rationale: Milk is an excellent source of calcium, which is crucial for maintaining bone health. Patients with osteoporosis often require increased calcium intake.

HESI Exit Question 264: A 25-year-old patient presents with weakness, muscle cramps, and an irregular heartbeat. Laboratory tests reveal low serum potassium levels. What condition is the patient likely experiencing due to potassium deficiency?

A) Scurvy B) Rickets C) Hypokalemia D) Osteomalacia

Correct Answer: C) Hypokalemia

Rationale: The patient’s weakness, muscle cramps, and irregular heartbeat are indicative of hypokalemia, a condition characterized by low serum potassium levels.

HESI Exit Question 265: A 55-year-old patient with a history of high blood pressure is advised to follow a low-sodium diet. Which of the following foods should the patient avoid due to its high sodium content?

A) Fresh fruits B) Canned soup C) Brown rice D) Lean protein

Correct Answer: B) Canned soup

Rationale: Canned soups are often high in sodium, and patients with high blood pressure should avoid them as part of a low-sodium diet.

HESI Exit Question 266: A 40-year-old patient with celiac disease is at risk of nutrient deficiencies due to malabsorption. Which nutrient is commonly deficient in individuals with celiac disease?

A) Vitamin D B) Vitamin C C) Vitamin B12 D) Folate

Correct Answer: A) Vitamin D

Rationale: Individuals with celiac disease are at risk of vitamin and mineral deficiencies due to malabsorption. Vitamin D deficiency is common and can lead to various health issues.

HESI Exit Question 267: A 50-year-old patient with a history of gallstones is advised to reduce dietary fat intake. Which type of dietary fat should the patient limit to prevent gallstone formation?

A) Saturated fat B) Monounsaturated fat C) Polyunsaturated fat D) Omega-3 fatty acids

Correct Answer: A) Saturated fat

Rationale: Saturated fat intake is associated with an increased risk of gallstone formation. Patients with a history of gallstones are advised to limit their saturated fat intake.

HESI Exit Question 268: A 70-year-old patient presents with confusion, dry skin, and brittle nails. Laboratory tests reveal low serum thyroid hormone levels. What nutrient deficiency is most likely contributing to these symptoms?

A) Vitamin B12 deficiency B) Iron deficiency C) Iodine deficiency D) Calcium deficiency

Correct Answer: C) Iodine deficiency

Rationale: The patient’s symptoms of confusion, dry skin, brittle nails, and low thyroid hormone levels are indicative of iodine deficiency, which is necessary for thyroid function.

HESI Exit Question 269: A 35-year-old patient with a history of kidney stones is advised to increase fluid intake. What type of fluid should the patient prioritize to reduce the risk of kidney stone recurrence?

A) Soda B) Coffee C) Water D) Fruit juice

Correct Answer: C) Water

Rationale: Water is the preferred fluid for individuals with a history of kidney stones as it helps dilute urine and reduce the risk of stone formation.

HESI Exit Question 270: A 28-year-old patient presents with easy bruising, bleeding gums, and a history of a limited diet without fresh fruits and vegetables. What nutrient deficiency is most likely contributing to these symptoms?

A) Vitamin A deficiency B) Vitamin K deficiency C) Vitamin E deficiency D) Vitamin B1 deficiency

Correct Answer: B) Vitamin K deficiency

Rationale: The patient’s symptoms of easy bruising and bleeding gums are indicative of vitamin K deficiency, which is essential for proper blood clotting.

HESI Exit Question 271: A 45-year-old patient with a history of gastric bypass surgery is at risk of nutrient deficiencies. Which nutrient absorption is primarily affected by this surgery?

A) Vitamin D B) Vitamin C C) Vitamin B12 D) Folate

Correct Answer: C) Vitamin B12

Rationale: Gastric bypass surgery can significantly impact the absorption of vitamin B12, leading to deficiency in some patients.

HESI Exit Question 272: A 20-year-old athlete presents with muscle weakness, fatigue, and muscle cramps. Laboratory tests reveal low serum calcium levels. What condition is the patient likely experiencing due to calcium deficiency?

A) Scurvy B) Rickets C) Hypocalcemia D) Osteomalacia

Correct Answer: C) Hypocalcemia

Rationale: The patient’s muscle weakness, fatigue, muscle cramps, and low serum calcium levels are indicative of hypocalcemia, a condition characterized by low calcium levels in the blood.

HESI Exit Question 273: A 60-year-old patient with a history of osteoarthritis is advised to include foods rich in antioxidants in their diet. Which of the following nutrients acts as an antioxidant and can help manage osteoarthritis symptoms?

A) Iron B) Vitamin C C) Calcium D) Vitamin D

Correct Answer: B) Vitamin C

Rationale: Vitamin C is an antioxidant that can help manage osteoarthritis symptoms by reducing oxidative stress in the joints.

HESI Exit Question 274: A 50-year-old patient with a history of coronary artery disease is advised to follow a heart-healthy diet. Which dietary component should the patient limit to reduce the risk of cardiovascular events?

A) Dietary fiber B) Saturated fat C) Omega-3 fatty acids D) Calcium

Correct Answer: B) Saturated fat

Rationale: Limiting saturated fat intake is recommended for patients with a history of coronary artery disease to reduce the risk of cardiovascular events.

HESI Exit Question 275: A 40-year-old patient with a history of diabetes is advised to monitor carbohydrate intake. Which type of carbohydrate should the patient prioritize for better blood sugar control?

A) Simple carbohydrates B) Complex carbohydrates C) Sugary beverages D) Processed carbohydrates

Correct Answer: B) Complex carbohydrates

Rationale: Complex carbohydrates, such as whole grains and legumes, are preferable for individuals with diabetes as they have a lower impact on blood sugar levels compared to simple or processed carbohydrates.

HESI Exit Question 276: A 55-year-old patient with a history of hypertension is advised to follow a low-sodium diet. Which of the following food items is typically high in sodium and should be limited in the patient’s diet?

A) Fresh fruits B) Fresh vegetables C) Canned soup D) Whole grains

Correct Answer: C) Canned soup

Rationale: Canned soups are often high in sodium, and patients with hypertension are advised to limit their sodium intake as part of a low-sodium diet.

HESI Exit Question 277: A 65-year-old patient with a history of diverticulitis is advised to increase dietary fiber intake. Which of the following foods is an excellent source of dietary fiber?

A) White rice B) White bread C) Apples D) Processed meats

Correct Answer: C) Apples

Rationale: Apples are a good source of dietary fiber, which can help prevent diverticulitis and improve digestive health.

HESI Exit Question 278: A 45-year-old patient presents with a fever, productive cough, and chest pain. Chest X-ray reveals a consolidation in the right lower lobe of the lung. Which of the following antibiotics is the most appropriate initial treatment for this patient’s community-acquired pneumonia?

A) Ciprofloxacin B) Azithromycin C) Vancomycin D) Amoxicillin/clavulanate

Correct Answer: B) Azithromycin

Rationale: Azithromycin is a suitable choice for the initial treatment of community-acquired pneumonia, especially when the patient does not require hospitalization.

HESI Exit Question 279: A 30-year-old sexually active patient presents with painful urination and urethral discharge. A Gram stain of the discharge reveals intracellular gram-negative diplococci. What is the most appropriate treatment for this patient’s infection?

A) Azithromycin B) Metronidazole C) Ciprofloxacin D) Doxycycline

Correct Answer: D) Doxycycline

Rationale: The presence of intracellular gram-negative diplococci on Gram stain is indicative of Neisseria gonorrhoeae infection. Doxycycline is an appropriate treatment option.

HESI Exit Question 280: A 60-year-old patient with a history of methicillin-resistant Staphylococcus aureus (MRSA) infection presents with cellulitis. Which antibiotic is most effective for treating MRSA-related skin and soft tissue infections?

A) Ciprofloxacin B) Amoxicillin/clavulanate C) Clindamycin D) Penicillin VK

Correct Answer: C) Clindamycin

Rationale: Clindamycin is an effective antibiotic for treating MRSA-related skin and soft tissue infections.

HESI Exit Question 281: A 55-year-old patient presents with recurrent episodes of genital herpes. What antiviral medication is commonly used to suppress the frequency and severity of genital herpes outbreaks?

A) Ciprofloxacin B) Metronidazole C) Acyclovir D) Doxycycline

Correct Answer: C) Acyclovir

Rationale: Acyclovir is a commonly used antiviral medication for suppressing the frequency and severity of genital herpes outbreaks.

HESI Exit Question 282: A 40-year-old patient presents with fever, chills, and a swollen, painful joint. Joint aspiration reveals synovial fluid with elevated white blood cell counts and crystals. What medication is commonly used to manage this patient’s acute gouty arthritis?

A) Ciprofloxacin B) Indomethacin C) Penicillin VK D) Amoxicillin/clavulanate

Correct Answer: B) Indomethacin

Rationale: Indomethacin is a nonsteroidal anti-inflammatory drug (NSAID) commonly used to manage acute gouty arthritis by reducing inflammation and pain.

HESI Exit Question 283: A 50-year-old patient presents with a red, warm, and swollen lower leg. The patient has a history of deep vein thrombosis (DVT). What medication is commonly used to treat this patient’s suspected cellulitis?

A) Ciprofloxacin B) Clindamycin C) Warfarin D) Vancomycin

Correct Answer: B) Clindamycin

Rationale: Clindamycin is commonly used to treat cellulitis in patients with a history of DVT, as it provides coverage against Streptococcus species and other common pathogens.

HESI Exit Question 284: A 65-year-old patient with a history of chronic obstructive pulmonary disease (COPD) presents with increased shortness of breath, increased sputum production, and purulent sputum. What antibiotic is typically prescribed as the first-line treatment for exacerbations of COPD?

A) Ciprofloxacin B) Azithromycin C) Amoxicillin/clavulanate D) Doxycycline

Correct Answer: C) Amoxicillin/clavulanate

Rationale: Amoxicillin/clavulanate is often prescribed as the first-line treatment for exacerbations of COPD due to its coverage of common respiratory pathogens.

HESI Exit Question 285: A 70-year-old patient with a history of chronic hepatitis C infection presents with cirrhosis and ascites. What medication is commonly used to manage ascites in patients with cirrhosis?

A) Metronidazole B) Spironolactone C) Vancomycin D) Penicillin VK

Correct Answer: B) Spironolactone

Rationale: Spironolactone is commonly used to manage ascites in patients with cirrhosis due to its diuretic properties and ability to reduce fluid retention.

HESI Exit Question 286: A 45-year-old patient presents with severe diarrhea and abdominal cramps. Stool studies reveal the presence of Clostridium difficile toxin. What antibiotic is the most appropriate treatment for this patient’s C. difficile infection?

A) Ciprofloxacin B) Metronidazole C) Vancomycin D) Doxycycline

Correct Answer: C) Vancomycin

Rationale: Vancomycin is the most appropriate treatment for C. difficile infection, especially when the infection is severe.

HESI Exit Question 287: A 35-year-old patient presents with a painful, swollen eye and difficulty seeing. Examination reveals a corneal ulcer. What topical antibiotic is commonly used to treat bacterial corneal ulcers?

A) Neomycin B) Polymyxin B C) Tobramycin D) Gentamicin

Correct Answer: C) Tobramycin

Rationale: Tobramycin is commonly used to treat bacterial corneal ulcers due to its broad-spectrum coverage of common eye pathogens.

HESI Exit Question 288: A 60-year-old patient with a history of end-stage renal disease is undergoing hemodialysis. The patient is at risk of bloodstream infections related to vascular access. What antibiotic is commonly used for prophylaxis in these patients?

A) Ciprofloxacin B) Azithromycin C) Vancomycin D) Cefazolin

Correct Answer: D) Cefazolin

Rationale: Cefazolin is commonly used for prophylaxis against bloodstream infections related to vascular access in patients undergoing hemodialysis.

HESI Exit Question 289: A 50-year-old patient presents with recurrent urinary tract infections (UTIs). What antibiotic is commonly used for long-term prophylaxis to prevent recurrent UTIs in women?

A) Ciprofloxacin B) Metronidazole C) Nitrofurantoin D) Doxycycline

Correct Answer: C) Nitrofurantoin

Rationale: Nitrofurantoin is commonly used for long-term prophylaxis to prevent recurrent UTIs in women due to its efficacy against urinary pathogens.

HESI Exit Question 290: A 70-year-old patient presents with fever, confusion, and a positive blood culture for Streptococcus pneumoniae. What antibiotic is the initial treatment of choice for this patient’s suspected pneumococcal bacteremia?

A) Ciprofloxacin B) Azithromycin C) Vancomycin D) Ceftriaxone

Correct Answer: D) Ceftriaxone

Rationale: Ceftriaxone is the initial treatment of choice for suspected pneumococcal bacteremia.

HESI Exit Question 291: A 40-year-old patient presents with a swollen, painful joint, and recent travel to a tropical region. What antibiotic is commonly used to treat this patient’s suspected septic arthritis caused by an arthropod-borne infection?

A) Ciprofloxacin B) Azithromycin C) Doxycycline D) Amoxicillin/clavulanate

Correct Answer: C) Doxycycline

Rationale: Doxycycline is commonly used to treat septic arthritis caused by arthropod-borne infections such as Lyme disease.

HESI Exit Question 292: A 55-year-old patient with diabetes presents with a painful, red, swollen, and warm foot. What antibiotic is the initial treatment of choice for this patient’s suspected diabetic foot infection?

A) Ciprofloxacin B) Azithromycin C) Vancomycin D) Piperacillin/tazobactam

Correct Answer: D) Piperacillin/tazobactam

Rationale: Piperacillin/tazobactam is often used as the initial treatment of choice for diabetic foot infections due to its broad-spectrum coverage.

HESI Exit Question 293: A 35-year-old patient presents with a painful, swollen finger and purulent discharge from a finger wound. What antibiotic is commonly used to treat this patient’s suspected paronychia?

A) Ciprofloxacin B) Azithromycin C) Amoxicillin/clavulanate D) Cephalexin

Correct Answer: D) Cephalexin

Rationale: Cephalexin is commonly used to treat paronychia, a soft tissue infection around the fingernail or toenail.

HESI Exit Question 294: A 50-year-old patient presents with a persistent cough, night sweats, and weight loss. Chest X-ray reveals a cavitary lung lesion. What is the most appropriate initial treatment for this patient’s suspected tuberculosis (TB)?

A) Ciprofloxacin B) Azithromycin C) Isoniazid and rifampin D) Amoxicillin/clavulanate

Correct Answer: C) Isoniazid and rifampin

Rationale: The most appropriate initial treatment for suspected TB is a combination of isoniazid and rifampin.

HESI Exit Question 295: A 5-year-old child presents with a high fever, sore throat, and difficulty swallowing. Examination reveals tonsillar exudate and tender cervical lymph nodes. What is the most appropriate antibiotic for the initial treatment of this patient’s suspected streptococcal pharyngitis?

A) Amoxicillin/clavulanate B) Cephalexin C) Azithromycin D) Ciprofloxacin

Correct Answer: A) Amoxicillin/clavulanate

Rationale: Amoxicillin/clavulanate is the first-line antibiotic for treating streptococcal pharyngitis in children due to its efficacy against Group A Streptococcus.

HESI Exit Question 296: A 2-month-old infant presents with fever, irritability, and poor feeding. Physical examination reveals bulging fontanelles. What antibiotic is the initial treatment of choice for this patient’s suspected bacterial meningitis?

A) Ciprofloxacin B) Amoxicillin C) Ceftriaxone D) Azithromycin

Correct Answer: C) Ceftriaxone

Rationale: Ceftriaxone is the initial treatment of choice for suspected bacterial meningitis in infants due to its broad-spectrum coverage, including against common pathogens causing meningitis.

HESI Exit Question 297: A 6-year-old child presents with a persistent cough, fever, and chest pain. Chest X-ray reveals a consolidation in the right lower lobe of the lung. What antibiotic is the most appropriate initial treatment for this child’s suspected community-acquired pneumonia?

A) Ciprofloxacin B) Azithromycin C) Amoxicillin D) Doxycycline

Correct Answer: B) Azithromycin

Rationale: Azithromycin is a suitable choice for the initial treatment of community-acquired pneumonia in children, especially when they do not require hospitalization.

HESI Exit Question 298: A 3-year-old child presents with ear pain and fever. Examination reveals a bulging and erythematous tympanic membrane. What antibiotic is the first-line treatment for this child’s acute otitis media?

A) Ciprofloxacin B) Amoxicillin/clavulanate C) Azithromycin D) Ceftriaxone

Correct Answer: B) Amoxicillin/clavulanate

Rationale: Amoxicillin/clavulanate is the first-line antibiotic for treating acute otitis media in children due to its efficacy against common causative pathogens.

HESI Exit Question 299: A 10-year-old child presents with fever, cough, and difficulty breathing. Chest X-ray reveals patchy infiltrates and atelectasis. What antibiotic is the most appropriate initial treatment for this child’s suspected mycoplasma pneumonia?

A) Ciprofloxacin B) Amoxicillin/clavulanate C) Azithromycin D) Doxycycline

Correct Answer: C) Azithromycin

Rationale: Azithromycin is effective in treating mycoplasma pneumonia, which is a common cause of atypical pneumonia in children.

HESI Exit Question 300: A 6-month-old infant presents with a barking cough, stridor, and respiratory distress. What medication is the initial treatment of choice for this infant’s suspected croup?

A) Albuterol B) Epinephrine C) Dexamethasone D) Amoxicillin/clavulanate

Correct Answer: B) Epinephrine

Rationale: Epinephrine is the initial treatment of choice for croup in infants and children due to its bronchodilator effects, which help alleviate airway obstruction.

HESI Exit Question 301: A 4-year-old child presents with a red, swollen eye and purulent discharge. What antibiotic is commonly used to treat this child’s suspected bacterial conjunctivitis?

A) Neomycin B) Polymyxin B C) Tobramycin D) Gentamicin

Correct Answer: C) Tobramycin

Rationale: Tobramycin is commonly used to treat bacterial conjunctivitis in children due to its broad-spectrum coverage of common eye pathogens.

HESI Exit Question 302: A 12-year-old child presents with a tick bite and erythema migrans. What antibiotic is the initial treatment of choice for this child’s suspected Lyme disease?

A) Ciprofloxacin B) Azithromycin C) Doxycycline D) Amoxicillin/clavulanate

Correct Answer: C) Doxycycline

Rationale: Doxycycline is the initial treatment of choice for Lyme disease in children over the age of 8 years due to its efficacy against Borrelia burgdorferi, the causative agent of Lyme disease.

HESI Exit Question 303: A 14-year-old adolescent presents with a painful vesicular rash on one side of the face. The rash is preceded by a tingling sensation. What antiviral medication is commonly used to treat this adolescent’s suspected herpes zoster (shingles)?

A) Acyclovir B) Amoxicillin C) Ceftriaxone D) Metronidazole

Correct Answer: A) Acyclovir

Rationale: Acyclovir is commonly used to treat herpes zoster (shingles) in adolescents to reduce the duration and severity of symptoms.

HESI Exit Question 304: A 5-year-old child presents with recurrent episodes of wheezing, coughing, and shortness of breath. What medication is commonly used as a long-term controller therapy for this child’s suspected asthma?

A) Albuterol B) Ipratropium bromide C) Montelukast D) Epinephrine

Correct Answer: C) Montelukast

Rationale: Montelukast is commonly used as a long-term controller therapy for asthma in children to help prevent asthma symptoms.

HESI Exit Question 305: A 2-year-old child presents with diarrhea, vomiting, and signs of dehydration. What oral rehydration solution (ORS) is recommended as the primary treatment for this child’s suspected acute gastroenteritis?

A) Water B) Apple juice C) Sports drinks D) Oral rehydration solution (ORS)

Correct Answer: D) Oral rehydration solution (ORS)

Rationale: ORS is recommended as the primary treatment for acute gastroenteritis in children to replace lost fluids and electrolytes.

HESI Exit Question 306: A 8-year-old child presents with a head lice infestation. What medication is commonly used to treat this child’s pediculosis?

A) Ciprofloxacin B) Azithromycin C) Permethrin D) Amoxicillin/clavulanate

Correct Answer: C) Permethrin

Rationale: Permethrin is commonly used to treat pediculosis (head lice) in children due to its effectiveness in killing lice and their eggs.

HESI Exit Question 307: A 7-year-old child presents with a skin rash characterized by pruritic, linear burrows with vesicles between the fingers and on the wrists. What medication is commonly used to treat this child’s suspected scabies infestation?

A) Ciprofloxacin B) Azithromycin C) Permethrin D) Doxycycline

Correct Answer: C) Permethrin

Rationale: Permethrin is commonly used to treat scabies infestations in children due to its efficacy in killing the mites responsible for the condition.

HESI Exit Question 308: A 3-month-old infant presents with a persistent diaper rash that is not responding to routine diaper changes and barrier creams. What antifungal medication is commonly used to treat this infant’s suspected diaper dermatitis caused by Candida?

A) Mupirocin B) Hydrocortisone C) Clotrimazole D) Neomycin

Correct Answer: C) Clotrimazole

Rationale: Clotrimazole is commonly used to treat diaper dermatitis caused by Candida in infants due to its antifungal properties.

HESI Exit Question 309: A 10-year-old child presents with itching and discomfort in the perianal area. Examination reveals the presence of pinworms. What medication is commonly used to treat this child’s suspected pinworm infection?

A) Albendazole B) Ivermectin C) Metronidazole D) Ceftriaxone

Correct Answer: A) Albendazole

Rationale: Albendazole is commonly used to treat pinworm infections in children due to its effectiveness in eliminating the parasites.

HESI Exit Question 310: A 6-year-old child presents with a fever, malaise, and a rash that started on the face and spread downward. What medication is commonly used to treat this child’s suspected varicella-zoster virus (chickenpox) infection?

A) Acyclovir B) Amoxicillin C) Ceftriaxone D) Metronidazole

Correct Answer: A) Acyclovir

Rationale: Acyclovir is commonly used to treat varicella-zoster virus (chickenpox) infections in children, especially in severe or complicated cases.

HESI Exit Question 311: A patient presents with severe chest pain that radiates down the left arm, shortness of breath, and diaphoresis. Which cardiac disorder is most likely responsible for these symptoms?
A) Mitral valve prolapse
B) Atrial fibrillation
C) Hypertension
D) Acute myocardial infarction (AMI)

Explanation: These symptoms are classic indicators of an AMI, commonly known as a heart attack. It occurs when blood flow to a part of the heart muscle is blocked, leading to chest pain, shortness of breath, and other symptoms. The other options are cardiac disorders but do not typically present with these specific symptoms.